Sunteți pe pagina 1din 73

Universidad Distrital Francisco Jos de Caldas - Anlisis de Seales y Sistemas - Marco A.

Alzate 1
Primera Clase. El papel de este curso en el programa curricular de ingeniera electrnica

Quienes toman este curso inician su sexto semestre de ingeniera electrnica, por lo que merecen
una gran felicitacin por acabar de superar la primera mitad de su carrera. Sin embargo, cuando se
les pregunta qu es la ingeniera electrnica?, casi nadie es capaz de ofrecer una definicin precisa
y clara de la profesin que escogi estudiar, a pesar de estar en un estado tan avanzado de sus
estudios. Sin embargo, recogiendo los elementos que entre todos empiezan a proponer tentativa y
tmidamente, se puede ir construyendo una aproximacin que, en muchos casos, termina con una
definicin parecida a la siguiente:

La ingeniera electrnica es una profesin (esto es, un conjunto de conocimientos,
habilidades y formas de enfrentar un tipo particular de problemas con el propsito de
facilitar la vida de las personas) que aplica los principios fsicos del electromagnetismo
y la mecnica cuntica para el diseo, construccin, operacin y mantenimiento de
estructuras, mquinas, aparatos y procesos, de manera que se conozca su
comportamiento bajo condiciones de operacin especficas, con niveles de seguridad
especficos y con costos mnimos. Hasta aqu, no se diferencia de la ingeniera elctrica.
Sin embargo, mientras la ingeniera elctrica utiliza estos principios con el propsito de
generar, convertir, distribuir y controlar energa, el ingeniero electrnico los utiliza con
el propsito de capturar, almacenar, transmitir y procesar informacin.

Preguntar por sus reas de inters y sus motivaciones para estudiar ingeniera electrnica y hacer
una lista:

Telecomunicaciones, ingeniera de computadores, sistemas de control, instrumentacin,
componentes y microelectrnica, bio-ingeniera, telemtica, conmutacin, redes de comunicaciones,
tecnologa para msica, video (cine, televisin, multimedia), etc.

Reconocer que todas esas reas de actuacin de la ingeniera electrnica obedecen a la definicin
dada y que en todas ellas la informacin es el objeto principal, la cual se representa mediante
seales electromagnticas (corrientes, voltajes, campos), aunque sean transducciones de otros tipos
de seales (presin, temperatura, intensidad de luz, etc.). Felicitarlos porque a partir de sptimo
semestre empiezan a estudiar esos temas que eran la motivacin original para empezar esta carrera
y preguntarles, entonces, qu los han puesto a estudiar hasta ahora? Hacer una lista:

Circuitos, electrnicas, fsicas y Matemticas! Algebra lineal, clculo diferencial, clculo
integral, clculo vectorial, variable compleja, ecuaciones diferenciales, Fourier, probabilidades y
estadstica Muy chvere! Felicitarlos nuevamente, ponerlos entonces a resolver un problema de
regla de tres y notar que muy pocos de ellos lo resuelven bien Qu pas? De qu sirvi estudiar
tanta matemtica que los volvi muy buenos en calcular integrales muy complejas y en resolver
ecuaciones diferenciales con derivada parciales de orden superior y condiciones de frontera, pero
los hizo olvidar cmo plantear un problema de regla de tres, lo cual ya saban hacer en cuarto de
primaria? Deben preocuparse por eso? Cun til ha sido, entonces, lo que han aprendido en los
primeros cinco semestres? Han aprendido a pensar como ingenieros? Qu implicaciones tendr en
su actitud como estudiantes de ingeniera electrnica el descubrir que, despus de cinco semestres,
no saban qu es la ingeniera electrnica y que los muchos cursos de matemticas avanzadas para
ingeniera slo les han servido para olvidar cmo formular problemas de regla de tres? Notar que si
el mismo problema se les propone en trminos de resistencias y corrientes en vez de obreros y
paredes, todos lo hubieran resuelto correctamente en muy breve tiempo han olvidado cmo
pensar por aprender cmo derivar? Parece que debimos desconectar el cerebro para aprender como
maquinitas a hacer procesos de carpintera matemtica.
Universidad Distrital Francisco Jos de Caldas - Anlisis de Seales y Sistemas - Marco A. Alzate 2

Pues bien, este curso conecta todo lo que hemos visto hasta ahora en una teora bsica que se
constituye en el fundamento de todas las reas de especialidad de la ingeniera electrnica (y de
todas las ingenieras y toda la ciencia fsica)! Por supuesto, debemos volver a conectar el cerebro
para darle sentido a todo lo que hemos estudiado, no como maquinitas de resolver ecuaciones
diferenciales, de calcular integrales o de invertir matrices, sino como ingenieros, con pensamiento
crtico, capaces de relacionar nuestro conocimiento previo con cualquier nuevo conocimiento,
mediante procesos lgicos de deduccin, inferencia o induccin, y con capacidad argumentativa
para describir nuestros procesos lgicos.

Figura 1. El anlisis de seales es la cintura del gran reloj de arena en la formacin de un ingeniero electrnico

Leer, analizar y discutir el syllabus del curso, que se encuentra en
http://www.udistrital.edu.co/wpmu/malzate. Indicar que all se dispondr de las tareas y de otros
recursos.












Comunica-
ciones
Control
Bioinge-
niera
Computa-
dores
Telemtica
Instrumen-
tacin
Compo-
nentes
Algeabra
lineal
Fourier
Clculo
diferencial
Clculo
integral Clculo
vectorial Ecuaciones
diferenciales
Variable
compleja
Anlisis de seales
Universidad Distrital Francisco Jos de Caldas - Anlisis de Seales y Sistemas - Marco A. Alzate 3
Segunda Clase. Definicin de seal

Presentar algunas seales para considerar qu es lo que vamos a estudiar durante el curso (Las
seales estn en http://www.udistrital.edu.co/wpmu/malzate). Notar que todas ellas corresponden a
la grfica de una magnitud con respecto a otra: Nmero de manchas en el sol graficada con respecto
a cada uno de los meses de un perodo de 250 aos, o la intensidad lumnica de un pixel con
respecto a su posicin en coordenadas (x,y), o el intervalo entre disparos de una neurona auditiva
con respecto al nmero de disparo. Cmo podremos asociar con semejante disparidad de seales
un nico modelo matemtico que nos sirva para construir una teora unificada de seales y
sistemas?



1750 1800 1850 1900 1950
0
50
100
150
200
250
Manchas del sol, cada mes, desde 1750
N

m
e
r
o

d
e

m
a
n
c
h
a
s
ao
0 0.5 1 1.5 2 2.5
x 10
-3
-0.2
-0.15
-0.1
-0.05
0
0.05
0.1
Seal de ecolocalizacin de un murcilago
tiempo en segundos
A
m
p
l
i
t
u
d
Universidad Distrital Francisco Jos de Caldas - Anlisis de Seales y Sistemas - Marco A. Alzate 4



0 0.2 0.4 0.6 0.8 1 1.2 1.4 1.6 1.8
-0.8
-0.6
-0.4
-0.2
0
0.2
0.4
0.6
0.8
Sonido de grillos
tiempo en segundos
A
m
p
l
i
t
u
d
-1.5 -1 -0.5 0 0.5 1 1.5
-1.5
-1
-0.5
0
0.5
1
1.5
Seal recibida por un modem V.29 a 9600 bps
Parte real
P
a
r
t
e

i
m
a
g
i
n
a
r
i
a
0 2 4 6 8 10 12 14 16 18 20
-1500
-1000
-500
0
500
1000
1500
Electrocardiograma de un adulto
Tiempo, en segundos
A
m
p
l
i
t
u
d
Universidad Distrital Francisco Jos de Caldas - Anlisis de Seales y Sistemas - Marco A. Alzate 5




0.1 0.105 0.11 0.115 0.12 0.125 0.13 0.135 0.14 0.145 0.15
60
80
100
120
140
160
180
200
220
240
Flujo de sangre en la aorta de un cordero
Tiempo, en segundos
A
m
p
l
i
t
u
d
0 0.5 1 1.5 2 2.5 3 3.5 4 4.5
-0.8
-0.6
-0.4
-0.2
0
0.2
0.4
0.6
0.8
Voz femenina
tiempo en segundos
A
m
p
l
i
t
u
d
0 0.5 1 1.5 2 2.5
x 10
5
0
1
2
3
4
5
6
7
8
9
Logaritmo de la longitud de los archivos en mi disco duro
Nmero de archivo
l
o
g
a
r
i
t
m
o

d
e
l

T
a
m
a

o

e
n

b
y
t
e
s
Universidad Distrital Francisco Jos de Caldas - Anlisis de Seales y Sistemas - Marco A. Alzate 6



Lena
50 100 150 200 250 300 350 400 450 500
50
100
150
200
250
300
350
400
450
500
2 4 6 8 10 12
x 10
4
0.02
0.04
0.06
0.08
0.1
0.12
0.14
0.16
0.18
Intervalos entre descargas de una neurona que responde a un estmulo auditivo
Descarga
I
n
t
e
r
v
a
l
o
,

e
n

s
e
g
u
n
d
o
s
Universidad Distrital Francisco Jos de Caldas - Anlisis de Seales y Sistemas - Marco A. Alzate 7


Figura 2. Algunas seales tpicas

A partir de estas seales de ejemplo, podemos discutir qu es una seal? Escuchar propuestas y
concluir algo as:

Una seal es una cantidad fsica que vara en el tiempo, en el espacio, o con respecto
a cualquier otra cantidad fsica independiente, de manera que en sus variaciones
hay codificada una informacin.

Esta definicin se ajusta bien a cada una de las seales vistas anteriormente, pero nos deja perplejos
porque no imaginamos cmo estudiar una teora que resulte comn para un voltaje en un circuito, o
para unos intervalos entre disparos sucesivos de una neurona auditiva, o para las variaciones de los
precios en la bolsa de valores. Por eso preferimos un modelo matemtico que se ajuste igualmente
bien a todas las seales anteriores:

50 100 150 200 250
1050
1100
1150
1200
1250
Indice S&P durante los das no feriados de 2010
da
V
a
l
o
r

d
e

c
i
e
r
r
e
0 100 200 300 400 500 600 700 800
-6
-4
-2
0
2
4
6
8
x 10
-5
Seales ssmicas
Universidad Distrital Francisco Jos de Caldas - Anlisis de Seales y Sistemas - Marco A. Alzate 8
Una seal se representa mediante una funcin con un dominio y un rango
especficos, x:DR.

Es fcil construir estos modelos para las seales vistas ( es el conjunto de los enteros, es el
conjunto de los reales):

a. Nmero de manchas en el sol cada mes desde enero de 1700 hasta diciembre de 1995,
Nmero de bits que llegan en perodos de 20 ms a un enrutador IP:
x:
b. Seal de eco-localizacin de un murcilago, Sonido de grillos, Seal recibida por un
mdem V.29 a 9600 bps, Electrocardiograma de un adulto, Electrocardiograma de perro
anestesiado, Flujo de sangre en la aorta de un cordero, Sonido en la cabina de un F-16,
Voz femenina:
x:
c. Imagen de Lena:
x:[0511] [0511] [0255] (o, con mayor generalidad, x: )
d. Movimiento ssmico:
x:
e. Intervalos entre descargas de una neurona que responde a un estmulo auditivo; Instante de
llegada de paquetes a un enrutador IP; ndice diario S&P 500 durante el 2010:
x:
f. Nmero de paquetes en el buffer de un enrutador en cada instante de tiempo:
x:

Las seales tendrn unidades de amplitud y el dominio podr ser cualquier conjunto al que
llamaremos tiempo, a pesar de que pueda representar cualquier otra cantidad (como en la imagen
de Lena o en la seal de los impulsos neuronales).

De otro lado, slo consideraremos dominios continuos (contenidos en ) o discretos (contenidos en
) y rangos continuos (contenidos en o , donde es el conjunto de los nmeros complejos) o
discretos (contenidos en ). En cada caso, las seales toman nombres particulares, como muestra la
siguiente Figura.

Por ejemplo, la voz femenina es una seal anloga, la seal compuesta por los intervalos entre
disparos consecutivos de una neurona es una seal muestreada, el nmero de manchas en el sol o el
nmero de llegadas a un enrutador en intervalos fijos de tiempo son seales digitales, y el nmero
de paquetes en el buffer de un enrutador en una red de computadores es una seal cuantizada. El
nico tipo de seales que podemos procesar con un computador es el de las seales digitales: Son
de tiempo discreto porque en cada posicin de memoria podemos guardar una muestra de la seal, y
son de amplitud discreta porque en cada posicin de memoria slo podemos guardar un nmero
finito de posibles valores debido al tamao en bits de la palabra almacenada. Cuando la amplitud se
representa con 8 16 bits, el fenmeno de la cuantizacin se hace apreciable. Pero cuando tenemos
longitud de palabras de 64 bits, podramos representar nmeros tan pequeos como 2
-63
(la dcima
parte del dmetro de un electrn) y tan grandes como 2
63
(del orden de las distancias entre clusters
de galaxias), dependiendo de cmo dividamos los bits para la mantisa y el exponente, por lo que
podemos imaginar que se trata de seales muestreadas o seales en tiempo discreto. Por eso la
Universidad Distrital Francisco Jos de Caldas - Anlisis de Seales y Sistemas - Marco A. Alzate 9
teora que desarrollaremos en este curso se refiere, principalmente, a los dos tipos de seales con
amplitud continua, a las que llamaremos seales en tiempo continuo (las seales anlogas) y
seales de tiempo discreto (las seales muestreadas).

Tiempo continuo Tiempo discreto
Amplitud
continua
Seal anloga

Seal muestreada


Amplitud
discreta
Seal cuantizada

Seal digital

Figura 3. Clasificacin de seales segn su dominio y su rango

























Universidad Distrital Francisco Jos de Caldas - Anlisis de Seales y Sistemas - Marco A. Alzate 10
Tercera clase. Definicin de sistema

Veamos que una seal es una cantidad fsica que vara en el tiempo, en el espacio, o con respecto a
cualquier otra cantidad fsica independiente, de manera que en sus variaciones hay codificada una
informacin. Tambin vimos que, como abstraccin matemtica de ese concepto, una seal se
representa mediante una funcin con un dominio y un rango especficos, x:D R. Por simplicidad,
a la variable que toma valores en el conjunto dominio le llamaremos tiempo y a la variable que
toma valores en el conjunto rango le llamaremos amplitud. La primera clasificacin de seales
que vimos fue de acuerdo con la naturaleza continua o discreta del tiempo y la amplitud: Seales
anlogas, seales cuantizadas, seales muestreadas o seales digitales.

Por ejemplo, considere la siguiente seal anloga:
1 1 si
( )
1 0 si
a
t t
x t
t
s

=

>


Podemos cuantizarla haciendo ( ) / 5 ( ) [2 1, 2 1) /10,
q
x t k si x t k k k = e + e . Tambin
podemos muestrearla haciendo [ ] ( /10 0.05),
s
x n x n n = e . Por ltimo, podemos digitalizarla
si la muestreamos y la cuantizamos, [ ] ( /10 0.05)
d q
x n x n = . Los resultados se muestran en la
siguiente figura.
Tiempo continuo Tiempo discreto
Amplitud
continua
Seal anloga, x
a
(t)

Seal muestreada, x
s
[n]

Amplitud
discreta
Seal cuantizada, x
q
(t)

Seal digital, x
d
[n]

Figura 4. Procesos de muestreo y cuantizacin

Ntese que las seales en tiempo discreto las hemos graficado con respecto a t
n
= n/10 0.05 y no
con respecto a n. Aunque normalmente se grafican con respecto al nmero de la muestra, n, hemos
-2 -1.5 -1 -0.5 0 0.5 1 1.5 2
0
0.1
0.2
0.3
0.4
0.5
0.6
0.7
0.8
0.9
1
-2 -1.5 -1 -0.5 0 0.5 1 1.5 2
0
0.1
0.2
0.3
0.4
0.5
0.6
0.7
0.8
0.9
1
-2 -1.5 -1 -0.5 0 0.5 1 1.5 2
0
0.1
0.2
0.3
0.4
0.5
0.6
0.7
0.8
0.9
1
-2 -1.5 -1 -0.5 0 0.5 1 1.5 2
0
0.1
0.2
0.3
0.4
0.5
0.6
0.7
0.8
0.9
1
Universidad Distrital Francisco Jos de Caldas - Anlisis de Seales y Sistemas - Marco A. Alzate 11
preferido este cambio en el eje del tiempo para poder superponer las cuatro seales y compararlas
con mayor claridad:

Figura 5. Una seal anloga, cuantizada, muestreada y digitalizada

Pero las seales, como cantidades fsicas medibles, existen en un ambiente particular en el que se
generan, se propagan, se almacenan, se transforman, etc. Ese ambiente que ejerce un proceso
transformador en una seal se conoce como sistema. En efecto, probablemente se trata de un
conjunto de elementos que interactan entre ellos para formar un todo, como una resistencia y un
condensador que forman un filtro, o un resorte y una masa que forman un oscilador, etc.

En el primer sistema, las seales son el voltaje de la fuente, la corriente en la malla, el voltaje en
cada componente, la potencia disipada en la resistencia, etc. En el segundo sistema, las seales son
la posicin, la velocidad y la aceleracin de la masa, la fuerza ejercida por el resorte, la fuerza de
friccin, etc. En cada caso, el sistema se expresa mediante unas leyes fsicas, que constituyen unas
relaciones matemticas entre unas seales y otras. De hecho, conociendo algunas de esas seales,
podemos especificar otras seales. Diramos que el sistema lo podemos representar (y ste es otro
modelo matemtico) como una relacin entre una seal de entrada y una seal de salida, que es otro
tipo de funcin que en matemticas se llama funcional, ya que su entrada es una funcin de un
conjunto de posibles funciones de entrada, y su salida es otra seal de un conjunto de posibles
seales de salida. As como una funcin convierte un elemento del dominio en un elemento del
rango, un funcional convierte una funcin de entrada en una funcin de salida.

Figura 6. Dos sistemas que procesan seales naturalmente

-2 -1.5 -1 -0.5 0 0.5 1 1.5 2
0
0.1
0.2
0.3
0.4
0.5
0.6
0.7
0.8
0.9
1


x
a
(t)
x
q
(t)
x
s
[n]
x
d
[n]
Universidad Distrital Francisco Jos de Caldas - Anlisis de Seales y Sistemas - Marco A. Alzate 12
En este curso diremos que un sistema es una forma de representar un proceso fsico que acepta una
seal de entrada (o varias) y la procesa para generar una seal de salida (o varias).


Figura 7. Representacin de un sistema como un funcional

Pero Cmo es que podemos construir un modelo matemtico vlido para representar seales de
presin y temperatura en sistemas como calderas que podamos utilizar para seales de radiacin en
sistemas como colisionadores de partculas? Consideremos los siguientes dos ejemplos:

Figura 8. Dos sistemas diferentes que conducen a una misma forma de abstraccin matemtica

En cada caso conocemos leyes de la naturaleza formuladas matemticamente que nos permiten
expresar unas seales en trminos de otras. Por ejemplo, sabemos que el voltaje de entrada es la
suma del voltaje de salida ms el voltaje en la resistencia, el cual es R veces la corriente, la cual es
C veces la variacin del voltaje de salida. Igualmente, sabemos que la fuerza total sobre el carro es
igual a su masa por la aceleracin (suponemos que el consumo de gasolina produce un cambio
despreciable en la masa del carro). Ahora consideremos el siguiente modelo abstracto:

Figura 9. Abstraccin matemtica para los dos sistemas anteriores

Ntese que si usamos v
i
(t) en vez de x(t), v
o
(t) en vez de y(t) y RC en vez de o, el modelo abstracto
podra ser una representacin del circuito RC. Pero si usamos F(t)/ en vez de x(t), v(t) en vez de
y(t) y M/ en vez de o, el modelo abstracto podra ser una representacin del sistema mecnico. De
hecho, estas relaciones fueron las que motivaron el desarrollo del computador anlogo a comienzos
de los 80s. En lo que a este curso respecta, entonces, {x(t)}
t
ser simplemente una seal de entrada
a un sistema que la procesa para obtener una seal de salida {y(t)}
t
, independientemente de que se
{x(t)eRango
x
, teDominio
x
} {y(t)eRango
y
, teDominio
y
}
i(t)
v
i
(t)
v
o
(t)
+
-
+
-
F(t)
v(t)
v(t)
M
0
0
0
( ) ( ) ( )
( ) ( ) ( )
( ) ( ) ( )
i R
i
i o
v t v t v t
v t v t R i t
d
v t v t RC v t
dt
= +
= +
= +
( ) ( ) ( )
1
( ) ( ) ( )
d
F t v t M v t
dt
M d
F t v t v t
dt


=
= +
1/o
}
x(t) y(t) E
+
_
( )
1
( ) ( ) ( )
( ) ( ) ( )
d
x t y t y t
dt
d
x t y t y t
dt
o
o
=
= +
Universidad Distrital Francisco Jos de Caldas - Anlisis de Seales y Sistemas - Marco A. Alzate 13
trate de voltajes, corrientes, fuerzas o velocidades: Modelos matemticos (funciones para las seales
y funcionales para los sistemas), como un concepto abstracto que podra representar cualquier
sistema fsico apropiado.

La clasificacin de seales segn su dominio sea continuo o discreto y segn su rango sea continuo
o discreto tambin se aplicar a los sistemas segn el tipo de seales que procesen. El primero de
los siguientes dos sistemas es un sistema en tiempo continuo, mientras el segundo es un sistema en
tiempo discreto:

Figura 10. Sistemas en tiempo continuo y en tiempo discreto

Aunque un sistema podra tener entradas en tiempo continuo y salidas en tiempo discreto (en cuyo
caso dentro del sistema existir al menos un muestreador) o viceversa (en cuyo caso dentro del
sistema existir al menos un interpolador).

Como un ejemplo inicial, considrese el sistema anterior en tiempo continuo, el cual representa un
sistema lineal de primer orden como el circuito RC o el automvil:
( ) ( ) ( )
d
x t y t y t
dt
o = +
Si consideramos un incremento de tiempo At en vez del diferencial dt, y consideramos slo
instantes de tiempo mltiplos de At, podramos aproximar el anterior sistema mediante
( ) (( 1) )
( ) ( )
y n t y n t
x n t y n t
t
o
A A
A = A +
A

Que podemos interpretar como un sistema en tiempo discreto:
( ) [ ] [ ] 1 [ 1], donde x n y n y n
t
o
| | | = + =
A


Figura 11. Aproximacin en tiempo discreto a los sistemas de la Figura 8

Como un ejemplo de sistemas ampliamente estudiados, consideremos un sistema de
comunicaciones en el que una fuente de informacin genera un mensaje que debe ser representado
en forma de seales fsicas para poder transmitirlo a travs de un canal, donde la seal transmitida
{x(t)e, te} {y(t)e, te}
{x[n]e, ne} {y[n]e, ne}
x[n] E y[n]
Retardo
y[n-1]
|
1
1 | +
Universidad Distrital Francisco Jos de Caldas - Anlisis de Seales y Sistemas - Marco A. Alzate 14
puede sufrir distorsiones, interferencias y ruido. La intencin es recuperar el mensaje original de la
manera ms oportuna y fidedigna posible. Si no el sistema entero, es claro que el transmisor, el
canal y el receptor corresponden al modelo matemtico que hemos estado considerando: Entra una
seal de un conjunto de posibles seales de entrada, la cual se transforma en una seal de un
conjunto de posibles seales de salida. Como las seales correspondientes se pueden modelar como
funciones del tiempo, cada subsistema resulta ser un funcional.

Figura 12. Modelo matemtico (abstraccin conceptual) de un sistema de comunicaciones

Otro ejemplo es un sistema de control realimentado, en el que se desea que un proceso particular (o
planta) produzca una respuesta satisfactoria de manera robusta, esto es, a pesar de cambios en el
ambiente. Para esto, se considera que la planta obedece a seales de control para producir la seal
de salida, esto es, el proceso a controlar es un sistema de procesamiento de seales. Entonces es
posible tomar la seal de salida para producir una seal realimentada que se compara con una seal
de referencia. Si son iguales, la planta est operando satisfactoriamente. Si no, un sistema adicional
usar la seal de diferencia como entrada para producir como salida los cambios necesarios en la
seal de control.

Figura 13. Modelo matemtico (abstraccin conceptual) de un sistema de control

Por ltimo, es IMPORTANTE notar que cuando decidimos utilizar modelos matemticos para
representar de manera simplificada alguna realidad compleja, estamos construyendo una
idealizacin que ser vlida slo en la medida en que el modelo capture los aspectos ms relevantes
de la realidad, y en la medida en que la realidad se ajuste con suficiente precisin a las suposiciones
del modelo. Esto implica, por ejemplo, que los valores de las seales se mantengan dentro de las
escalas de validez del modelo, o que los parmetros que describen a los componentes se encuentren
suficientemente cerca de los valores usados en el modelo. En general, una caracterstica
fundamental del ingeniero es su capacidad de determinar el alcance de la validez de los modelos
que utiliza, asegurndose que en el proceso de anlisis o diseo que adelanta siempre se cumplan
las condiciones y suposiciones en las que se bas el desarrollo de su modelo matemtico.


Fuen-
te
Trans-
ductor
Trans-
misor
Canal Receptor
Trans-
ductor
Desti-
no
Ruido,
interferencia
y distorsin
Mensaje
Seal de
entrada
Seal
transmitida
Seal
recibida
Seal de
salida
Mensaje
Elemento
de control
Proceso a
controlar
Lazo de
realimentacin
Seal de
referencia
Compa-
racin
Seal de
diferencia
Seal de
control
Seal de
salida
Seal re-
alimentada
Universidad Distrital Francisco Jos de Caldas - Anlisis de Seales y Sistemas - Marco A. Alzate 15
Primera tarea

En esta tarea compararemos la validez de la aproximacin entre los dos sistemas en tiempo
continuo y en tiempo discreto que vimos en la tercera clase.

( ) [ ] [ ] 1 [ 1], donde
d d d
x n y n y n
t
o
| | | = + =
A

En esta aproximacin, si x
d
[n] es la n-sima muestra de x(t), x
d
[n]=x(nAt), esperaramos que y
d
[n] se
pareciera a la n-sima muestra de y(t), y
d
[n]~y(nAt). Para verificar la validez de esto, suponga que
queremos representar el circuito RC con o = RC = 10 ms y un incremento discreto de tiempo At = 1
ms, de manera que el parmetro | del sistema en tiempo discreto es | = 10. Suponga que la seal de
entrada al sistema continuo es un paso de amplitud 1 y 30 ms de duracin, como se muestra a
continuacin:

(a) Encuentre analticamente la salida del sistema en tiempo continuo, y(t), suponiendo que y(0
-
) =
0, y grafquela para t entre 0 y 80 ms.
(b) Obtenga x
d
[n] tomando muestras de x(t) cada At = 1 ms, x
d
[n] = x(n/1000), calcule la salida del
sistema en tiempo discreto, y
d
[n], suponiendo que y
d
[-1]=0, y grafquela para n entre 0 y 80.
(c) Calcule el error cuadrado promedio (MSE Mean Square Error) en el rango [0, 0.08]
( )
80
2
0
1
( / 1000) [ ]
81
d
n
MSE y n y n
=
=

.
(d) Repita los puntos (b) y (c) con At = 0.2 ms (x
d
[n] = x(n/5000), | = 50, n entre 0 y 400,
( )
400
2
0
1
( / 5000) [ ]
401
d
n
MSE y n y n
=
=

.
Escriba un breve prrafo de conclusiones.


1/o
}
x(t) y(t) E
+
_
( )
1
( ) ( ) ( )
( ) ( ) ( )
d
x t y t y t
dt
d
x t y t y t
dt
o
o
=
= +
x
d
[n] E y
d
[n]
Retardo
y
d
[n-1]
|
1
1 | +
t
x(t)
0 30 ms
1 V
0 V
Universidad Distrital Francisco Jos de Caldas - Anlisis de Seales y Sistemas - Marco A. Alzate 16
Cuarta clase. Potencia y energa

(Discusin de la primera tarea)

Como, en nuestro modelo conceptual abstracto, las seales tienen unidades de amplitud arbitrarias,
podemos definir otras cantidades de inters asociadas con una seal. Por ejemplo, la potencia
instantnea en una resistencia de R ohmios a la que se aplica una seal de v(t) voltios es |v(t)|
2
/R
watios, proporcional al cuadrado del valor absoluto de la seal. Como las unidades no son de inters
para nuestro modelo terico, definiremos la potencia de una seal x(t) como |x(t)|
2
. En el caso de
una seal de tiempo discreto, x[n], la potencia instantnea de la seal se definir como |x[n]|
2
. A
partir de esta definicin, podemos definir tambin la energa en un intervalo de tiempo, la potencia
promedio en un intervalo, la energa total y la potencia promedio total:

Tiempo continuo Tiempo discreto
Potencia instantnea
2
( ) ( )
x
P t x t =
| |
2
[ ]
x
P n x n =
Energa en un intervalo 2
( , ) ( )
t
x
t
E t t x s ds
t
t

=
}

2
[ , ] [ ]
n
x
k n N
E n N n x k
=
=


Potencia promedio en un
intervalo
2 1
( , ) ( )
t
x
t
P t t x s ds
t
t
t

=
}

2 1
[ , ] [ ]
1
n
x
k n N
P n N n x k
N
=
=
+


Energa total 2
( )
x
E x t dt

=
}

2
[ ]
x
n
E x n

=
=


Potencia promedio total
2 1
lim ( )
2
T
x
T T
P x t dt
T

=
}

2 1
lim [ ]
2 1
N
x
N
n N
P x n
N

=
=
+


Figura 14. Definiciones abstractas de potencia y energa

Con estas definiciones podemos considerar una nueva clasificacin de seales: Una seal x(t) o x[n]
se conoce como seal de energa si su energa total es finita: 0 < E
x
< . Una seal x(t) o x[n] se
conoce como seal de potencia si su potencia promedio total es finita: 0 < P
x
< . En otro caso, la
seal no es ni una ni otra. Por supuesto, dadas las relaciones entre energa total y potencia promedio
total, la condicin E
x
< implica P
x
= 0, mientras que la condicin P
x
> 0 implica E
x
= . Es decir,
la clasificacin como seal de energa o seal de potencia es excluyente.

Estas definiciones, aunque inspiradas en los conceptos fsicos de potencia y energa, parecen
arbitrarias para nuestro modelo abstracto, adimensional y puramente matemtico. Sin embargo,
notaremos cmo se van convirtiendo en definiciones fundamentales para especificar los tipos de
procesos que podemos aplicar a cada tipo de seal. Para dar una indicacin inicial, ntese que una
seal peridica es una seal de potencia, mientras que una seal de soporte compacto, esto es,
idnticamente igual a cero por fuera de un intervalo finito, es una seal de energa (siempre y
cuando ambas sean diferentes de la seal idnticamente cero y acotadas). A la primera le
aplicaremos la serie de Fourier y a la segunda le aplicaremos la transformada de Fourier.

Universidad Distrital Francisco Jos de Caldas - Anlisis de Seales y Sistemas - Marco A. Alzate 17
A manera de ejemplo, ntese que la seal x[n] = 4 neZ no es una seal de energa porque
2
[ ] 16
x
n n
E x n

= =
= = =

, pero s es una seal de potencia porque, como todas las muestras
tienen una potencia instantnea 16, la potencia promedio por muestra en cualquier intervalo de
tiempo es 16:
2 1 1 2 1
lim [ ] lim 16 lim16 16
2 1 2 1 2 1
N N
x
N N N
n N n N
N
P x n
N N N

= =
+
= = = =
+ + +


La seal x(t) = t te no es de potencia ni de energa pues es fcil ver que ambas cantidades son
infinitas:
2 2
2 3 2
0
0
1 1 1 1 1
lim ( ) lim lim lim lim
2 2 3 3
T
T T T
x
T T T T T T T
P x t dt t dt t dt t T
T T T T

( = = = = = =
} } }

Claro, si P
x
es infinita, con mayor razn lo ser E
x
. Por ltimo, consideremos la seal x(t) = exp(-|t|):
2
2 2 2 2
0
0 0
1
( ) 2 2 0 ( 1) 1
2
t t t t
x
E x t dt e dt e dt e e




(
( = = = = = = =
(

} } }

Por lo que se trata de una seal de energa. Claro, su potencia promedio es cero a pesar de que la
seal toma un valor mayor que cero para cualquier instante de tiempo.


























Universidad Distrital Francisco Jos de Caldas - Anlisis de Seales y Sistemas - Marco A. Alzate 18
Quinta clase. Introduccin a Matlab

Matlab es un programa de computacin cientfica especialmente dirigido a aplicaciones de
ingeniera. Los principales objetos con que se interacta en matlab son las sentencias, las variables,
las matrices, las grficas y los scripts. Como introduccin a matlab, en esta clase veremos estos
cinco objetos rpidamente, reconociendo que, como todo, matlab slo se llega a conocer despus de
trabajar intensamente con l.

Al ejecutar matlab (por ejemplo, si se usa Windows de microsoft, haciendo doble click en el
cono correspondiente), aparece un ambiente de programacin y ejecucin con algunos mens
descolgantes e conos que tambin puede incluir una ventana con la lista del contenido del
directorio de trabajo, una ventana con la lista de las variables en la memoria (o workspace -espacio
de trabajo-), una ventana con la lista de los comandos recientes, etc. Pero la parte principal de la
pantalla es una ventana donde se introducirn comandos interactivos, inmediatamente despus del
prompt >>.

Figura 15. Ambiente de interaccin con Matla

Por ejemplo, una sentencia tpica es la asignacin de una matriz 2x2 a una variable de Matlab:
>> A = [3 2; 1 7]
A =
3 2
1 7
La matriz A que se acaba de crear aparece automticamente despus de oprimir ENTER, con lo que
reporta el resultado de la operacin. Si la sentencia se hubiera terminado con un punto-y-coma, (;),
el despliegue de A se hubiera eliminado. Los nombres de las variables empiezan con una letra y
pueden contener nmeros, maysculas y minsculas, y el smbolo _. Tenga en cuenta que Matlab
Directorio actual
Contenido del
directorio
indicado
Informacin de
las variables en
memoria
Informacin
del archivo
seleccionado
Listado de los
comandos
introducidos en
la ventana de
comandos
Ventana de
comandos para
interaccin con
Matlab
Botn de inicio
Universidad Distrital Francisco Jos de Caldas - Anlisis de Seales y Sistemas - Marco A. Alzate 19
distingue entre maysculas y minsculas, de manera que las variables a y A, por ejemplo, son
variables diferentes:
>> A = [3 2; 1 7];
>> a = 3;
>> B = a*A
B =
9 6
3 21
En sentencias como las anteriores se pueden utilizar los operadores matemticos usuales (+, -, *, /),
funciones ya predefinidas (por ejemplo sin(), cos(), tan(), asin(), acos(), atan(),
abs(), sqrt(), imag(), real(), conj(), log(), log2(), log10(), exp(), etc),
y algunos nombres de variables predefinidos como pi (= t), inf (= ), y j (= \(-1) ):
>> t = 0.125; f = 1000; phi = pi/8;
>> angulo = 2*pi*f*t + phi;
>> a = cos(angulo) + j*sin(angulo)
a =
0.9239 + 0.3827i
Aunque hay una ventana que muestra las variables en el workspace, esta informacin tambin se
puede obtener con la instruccin whos:
>> whos
Name Size Bytes Class Attributes
A 2x2 32 double
B 2x2 32 double
a 1x1 16 double complex
angulo 1x1 8 double
f 1x1 8 double
phi 1x1 8 double
t 1x1 8 double
La instruccin clear borra todas las variables de la memoria:
>> clear
>> whos
Una variable importante es ans, que se refiere al ltimo resultado obtenido:
>> 2*pi
ans =
6.2832
>> sqrt(ans)
ans =
2.5066
La precisin con que se muestran los resultados es diferente a la precisin con que se almacenan,
pues cada escalar se almacena con 64 bits:
>> format long
>> 2*pi
ans =
6.283185307179586
>> sqrt(ans)
ans =
2.506628274631000
Universidad Distrital Francisco Jos de Caldas - Anlisis de Seales y Sistemas - Marco A. Alzate 20
Por supuesto, el formato de salida no afecta los clculos, que siempre se hacen con precisin
double.

El nombre de Matlab es una contraccin de Matrix Laboratory pues, en efecto, la unidad
computacional bsica es la matriz. Ntese, por ejemplo, que en el listado de variables en memoria
anterior, a, angulo, f, phi y t son matrices de 11 en vez de escalares. Como se mostr en el
primer ejemplo, una matriz se introduce entre parntesis cuadrados [], separando las filas mediante
punto-y-coma (;) y separando los elementos de cada fila mediante coma (,) o espacio. Por
ejemplo, una manera de introducir la matriz
0.8
log( 1) sin( / 4) cos( / 6)
2 3
arcsin(0.2) (1,1) 1
A j e
t t
|
(
(
=
(
(


Podra ser mediante la siguiente instruccin:
>> A = [log(-1) sin(pi/4) cos(pi/6);
-2j sqrt(3) exp(0.8); asin(0.2), beta(1,1), 1]
A =
0 + 3.1416i 0.7071 0.8660
0 - 2.0000i 1.7321 2.2255
0.2014 1.0000 1.0000
Ntese que podemos usar mltiples lneas o no, y que podemos separar los elementos de una misma
fila mediante coma o mediante espacio. Cuando operamos con matrices podemos sumarlas,
restarlas, multiplicarlas, invertirlas, transponerlas, elevarlas a una potencia, etc. Claro, en estos
casos necesitamos que las dimensiones de las matrices sean compatibles. En el siguiente ejemplo, A
es 23 y B es 32, por lo que no se pueden sumar, pero la transpuesta de B s se puede sumar con A
(el apstrofe se refiere a la transpuesta conjugada que, en este caso en que Be
32
, resulta la misma
transpuesta).
>> A = [1 2 3; 4 5 6];
>> B = [1 2; 3 4; 5 6];
>> A+B
??? Error using ==> plus
Matrix dimensions must agree.
>> A+B'
ans =
2 5 8
6 9 12
Claro, A y B s se pueden multiplicar:
>> C=A*B
C =
22 28
49 64
Resultando una matriz cuadrada que se puede elevar al cuadrado:
>> C^2
ans =
1856 2408
4214 5468
Universidad Distrital Francisco Jos de Caldas - Anlisis de Seales y Sistemas - Marco A. Alzate 21
Pero si queremos construir una matriz en donde cada uno de los elementos de C est elevado al
cuadrado, precedemos el operador con un punto indicando la naturaleza elemento a elemento de la
operacin:
>> C.^2
ans =
484 784
2401 4096
Obsrvese cun fcil es calcular el producto interno o el producto externo entre dos vectores:
>> x = [1; 2; 3];
>> y = [4; 5; 6];
>> interno = x'*y
interno =
32
>> externo = x*y'
externo =
4 5 6
8 10 12
12 15 18
Claro, el producto interno x'*y es la suma de los productos de los componentes de cada vector:
>> sum(x.*y)
ans =
32
Aprovechemos para verificar que el producto de dos matrices es la transpuesta del producto de las
transpuestas:
>> y*x'
ans =
4 8 12
5 10 15
6 12 18

Si tenemos un sistema lineal de ecuaciones,
3x + 2y + z = 1 2x y + 3z = 2 -x + 4y 2z = 3
Se puede encontrar su solucin de manera muy simple:
>> inv([3 2 1; 2 -1 3; -1 4 -2])*[1; 2; 3]
ans =
-1.2857
1.4286
2.0000
donde inv(A) se refiere a la inversa de la matriz A. El resultado se puede verificar fcilmente:
>> [3 2 1; 2 -1 3; -1 4 -2]*[-1.2857143; 1.4285714; 2]
ans =
1.0000
2.0000
3.0000
Los componentes individuales de un vector, una matriz o cualquier otro arreglo, se referencian con
el nombre del arreglo sub-indicado con la posicin del componente que nos interesa, teniendo en
cuenta que el ndice del primer elemento es 1 (no cero como en muchos lenguajes de
programacin):
Universidad Distrital Francisco Jos de Caldas - Anlisis de Seales y Sistemas - Marco A. Alzate 22
>> A = [3 2 1; 6 5 4]
A =
3 2 1
6 5 4
>> A(2,1)
ans =
6
>> A(1,3)
ans =
1
De hecho, podemos extraer toda una fila o una columna usando dos-puntos (:) para referirnos a
todos los ndices correspondientes:
>> A(2,:)
ans =
6 5 4
>> A(:,2)
ans =
2
5
Ms an, la notacin dos-puntos (:) nos permite describir rangos de subndices:
>> A=[1 4 3 5 1; 2 5 1 2 6; 0 2 7 1 8; 3 1 4 2 9]
A =
1 4 3 5 1
2 5 1 2 6
0 2 7 1 8
3 1 4 2 9
>> A(2:3,2:4)
ans =
5 1 2
2 7 1
En estos casos, el subndice end puede ser de gran utilidad, pues se refiere al ltimo subndice en
la dimensin correspondiente:
>> A(3:end,4:end)
ans =
1 8
2 9
Una forma sencilla de introducir un vector fila es a travs de esta misma notacin basada en dos-
puntos (:)
>> x = 0:0.5:3
x =
0 0.5000 1.0000 1.5000 2.0000 2.5000 3.0000
En general x = xi : dx : xf define un vector fila que empieza con x(1)=xi, y para el cual el
elemento n+1 es igual al elemento n incrementado en dx, x(n+1) = x(n)+dx, para n=1,2,...,
de manera que x(end) s xf y x(end)+dx > xf. Se leera como desde xi hasta xf en
incrementos de dx. Si se omite el incremento, se asume que vale uno:
>> n=-3:5
n =
-3 -2 -1 0 1 2 3 4 5
Universidad Distrital Francisco Jos de Caldas - Anlisis de Seales y Sistemas - Marco A. Alzate 23
A manera de ejemplo, suponga que queremos evaluar una seal en tiempo continuo,
| |
3
( ) sin(8 ) , 0,1
t
x t t e t t

= e
Claro, el intervalo unitario es imposible de almacenar en un computador con memoria finita (an en
un computador con memoria infinita!), por lo que conviene tomar un vector de muestras
suficientemente densas en el tiempo:
>> t=0:0.005:1;
>> x=sin(8*pi*t).*exp(-3*t);
Con lo que hemos evaluado x(t) en 201 puntos del intervalo unitario. Ntese que, en la memoria de
trabajo de matlab, x es una seal muestreada, no una seal en tiempo continuo. Sin embargo, como
est densamente muestreada, podemos graficarla interpolando mediante lneas rectas entre las
muestras, para lo cual usamos la instruccin plot, dndonos la sensacin de continuidad:
>> plot(t,x)


Figura 16. Resultado de las instrucciones de matlab
t=0:0.005:1; x=sin(8*pi*t).*exp(-3*t); plot(t,x)

En el men de conos de la ventana de la grfica hay una pequea lupa con la que podemos ver con
mayor detalle una porcin de la figura para notar el tipo de interpolacin que hace la instruccin
plot:
Universidad Distrital Francisco Jos de Caldas - Anlisis de Seales y Sistemas - Marco A. Alzate 24

Figura 17. Zoom de la figura 16, donde se nota la interpolacin hecha por la instruccin plot

Por supuesto tratndose de seales muestreadas, es mejor utilizar una representacin grfica que
facilite esta interpretacin. Esto se consigue con la instruccin stem. Por ejemplo, ahora tomaremos
41 muestras de la seal anterior:
>> n = 0:40;
>> y=sin(8*pi*(n/40)).*exp(-3*(n/40));
>> stem(n,y)

Figura 18. Grfica de seales en tiempo discreto mediante stem

Es posible comparar ambas grficas ya que la instruccin hold on permite graficar sobre las
figuras previamente creadas. Claro, la seal muestreada no se debe graficar con respecto al nmero
de muestra sino con respecto al instante correspondiente de cada muestra:
>> t=0:0.005:1;
>> x=sin(8*pi*t).*exp(-3*t);
>> plot(t,x,r)
>> hold on
>> n = 0:40;
>> y=sin(8*pi*(n/40)).*exp(-3*(n/40));
>> stem(n/40,y)
0 5 10 15 20 25 30 35 40
-0.6
-0.4
-0.2
0
0.2
0.4
0.6
0.8
1
Universidad Distrital Francisco Jos de Caldas - Anlisis de Seales y Sistemas - Marco A. Alzate 25
Ntese que, por claridad, trazamos la seal continua en color rojo mediante el parmetro adicional
r del comando plot.

Figura 19. Despliegue simultneo de la seal en tiempo continuo y en tiempo discreto

Claramente, una de las principales caractersticas de Matlab es su excelente capacidad grfica y la
facilidad con que se utiliza dicha capacidad. Como para darnos una breve idea de las muchas
posibilidades, podemos usar el comando help, que despliega informacin de ayuda sobre otros
comandos de matlab. Ejecute los siguientes comandos y disfrute viendo las instrucciones que
matlab tiene para presentar grficas de muchos tipos diferentes:
>> help matlab\graph2d
>> help matlab\graph3d
>> help matlab\specgraph

Hasta ahora toda la interaccin que hemos tenido con matlab ha sido a travs de la ventana de
comandos, pero este modo de interaccin slo permite sesiones cortas y no repetitivas.
Afortunadamente, matlab permite editar, almacenar e invocar archivos de comandos donde se puede
agrupar un gran nmero de sentencias relacionadas para ser utilizadas como una nica sentencia en
la ventana de comandos. Estos archivos son archivos tipo .m ya que su nombre tiene esta
extensin. Existen dos tipos de archivos .m : Los archivos script (guin o libreto) son, simplemente,
una secuencia de comandos que se almacenan desde un editor de texto para que matlab los ejecute
como si fueran introducidos desde la ventana de comandos. Estos archivos pueden incluir llamadas
a otros scripts, pero debe tenerse cuidado porque trabajan con las variables del workspace, lo cual
puede ser muy conveniente para interactuar con ellos desde la ventana de comandos, pero tambin
puede producir interacciones no intencionales a travs de estas variables. El otro tipo de archivos .m
son los archivos function (funcin), los cuales pueden aceptar variables de entrada y pueden
producir variables de salida, aunque sus variables internas no hacen parte del workspace. Para
terminar este tutorial utilizaremos solamente archivos script, y dejaremos al lector la consideracin
de los archivos function (use >> help function en la lnea de comandos).

Por ejemplo, si queremos hacer un archivo que grafique un segundo de una seal seno con
frecuencia dada, podemos usar el editor de matlab as:

>> edit FiguraSeno
0 0.1 0.2 0.3 0.4 0.5 0.6 0.7 0.8 0.9 1
-0.6
-0.4
-0.2
0
0.2
0.4
0.6
0.8
1
Universidad Distrital Francisco Jos de Caldas - Anlisis de Seales y Sistemas - Marco A. Alzate 26
Con lo que creamos el archivo FiguraSeno.m en el directorio de trabajo y entramos al editor de
texto para digitarlo. En el editor simplemente escribimos los comandos a ejecutar (aunque no los va
a ejecutar inmediatamente, pues no estamos en la ventana de comandos):


Figura 20. Edicin de un archivo script

Despus de guardar el archivo (usando el cono del diskette o las opciones de men File/Save, o la
tecla rpida CTRL-S), podemos regresar a la ventana de comandos:

>> help FiguraSeno
FiguraSeno.m
Script para dibujar la seal x(t) = sin(2*pi*f*t) en el intervalo
unitario [0, 1]. El valor de f debe existir en el espacio de trabajo
antes de invocar este script.

Como se puede apreciar, los comentarios iniciales de un script aparecen cuando se pide ayuda sobre
el nuevo comando. En este caso, la ayuda es implacable: Habr problemas si no definimos
primero la variable f:

>> FiguraSeno
??? Undefined function or variable 'f'.
Error in ==> FiguraSeno at 7
x = sin(2*pi*f*t);

Mejor seguir las instrucciones:

>> f = 10; FiguraSeno

Ntese las etiquetas en la grfica (xlabel, ylabel, title). Con ellas aprovechamos para
ilustrar pasos bsicos del manejo de cadenas de caracteres cmo, por ejemplo, la inclusin de un
valor numrico dentro del arreglo de caracteres que conforman el ttulo (mediante la funcin
num2str que convierte un dato numrico en una cadena de caracteres).
Universidad Distrital Francisco Jos de Caldas - Anlisis de Seales y Sistemas - Marco A. Alzate 27

Figura 21. Resultado del script FiguraSeno fon f=10

Por ltimo, veamos cmo sera un script para ayudar a resolver la primera tarea

% Tarea1.m
% Resuelve el primer punto de la primera tarea
% dt debe estar definida en el workspace antes de invocar este script
% dt es el tiempo de muestreo para el sistema en tiempo discreto
%
% Toma suficientes muestras en el tiempo continuo de x(t) y de y(t)
t = -0.01:0.00002:0.08; % Toma muestras cada 20 us
xt = (t>=0).*(t<=0.03); % Seal de entrada
yt = xt.*(1-exp(-100*t)) + (1-exp(-3))*(t>0.03).*exp(-(100*t-3)); % Seal de salida
plot(t,xt,'b-',t,yt,'r-') % Grafica la salida en tiempo continuo
% Ahora toma las muestras para el sistema en tiempo discreto
n = floor(-0.01/dt):floor(0.08/dt); % tiempo discreto
a = 0.01; % parmetro alpha
b = a/dt; % parmetro beta
r = b/(b+1); % parmetro ro
n0 = floor(0.03/dt); % instante en el que x[n] cambia de uno a cero
yd = (n>=0).*(n<=n0).*(1-r.^(n+1)) + (n>n0).*(r.^(n-n0) - r.^(n+1));
hold on
stem(n*dt,yd,'k.')
xlabel('tiempo en segundos')
ylabel('Amplitud')
title(['Seales relacionadas con la tarea 1 cuando dt = ' num2str(dt)])
% Breve extensin para calcular el error en el intervalo [0 0.08]
t = n*dt;
yi = (t>=0).*(t<=0.03).*(1-exp(-100*t)) + (1-exp(-3))*(t>0.03).*exp(-(100*t-3));
I = find(n>=0); N = length(I);
MSE = sum((yi(I) - yd(I)).^2)/N

Ntese el uso de relaciones lgicas para definir rangos de validez de las expresiones. Por ejemplo,
cuando hacemos xt = (t>=0).*(t<=0.03), estamos multiplicando punto a punto una seal que
vale uno slo cuando t es no negativa con otra seal que vale uno slo cuando t es menor o igual a
30 ms. El producto es uno slo cuando 0 s t s 0.03, que era exactamente la definicin de x(t) en la
0 0.1 0.2 0.3 0.4 0.5 0.6 0.7 0.8 0.9 1
-1
-0.8
-0.6
-0.4
-0.2
0
0.2
0.4
0.6
0.8
1
Tiempo en segundos
A
m
p
l
i
t
u
d
x(t) = sin(2*pi*f*t), f=10
Universidad Distrital Francisco Jos de Caldas - Anlisis de Seales y Sistemas - Marco A. Alzate 28
tarea. Tambin ntese el uso de las funciones floor, find y sum (use help para ver sus
descripciones).

Figura 22. Resultado del script Tarea1.m con dt = 1 ms

El siguiente script, que invoca al anterior, nos permitira calcular el error cuadrado medio en el
intervalo [0, 0.08] para diferentes valores de dt:

for i=1:20
dt = (49*i - 30)/190000; % de 0.1 ms hasta 5 ms en 20 pasos
Tarea1
mse(i) = MSE;
d(i) = dt;
clear I MSE N a b dt n n0 r t x yd yi yt
end
close all
semilogy(d,mse)
xlabel('Intervalo de muestreo en segundos')
ylabel('Error cuadrado medio')
title('MSE de la aproximacin en el intervalo [0, 0.08]')

Figura 23. Error al aproximar el sistema en tiempo continuo por el sistema en tiempo discreto de la tarea 1

Ntese el uso de la instruccin clear para borrar algunas o todas las variables (algo que conviene
hacer al iniciar un nuevo proceso en el workspace para evitar las interacciones insospechadas de las
que hablamos anteriormente), la instruccin close para cerrar las ventanas grficas que estn
abiertas, y la instruccin semilogy para usar una escala logartmica en el eje vertical (use help
para conocer ms sobre estas instrucciones).
-0.01 0 0.01 0.02 0.03 0.04 0.05 0.06 0.07 0.08
0
0.1
0.2
0.3
0.4
0.5
0.6
0.7
0.8
0.9
1
tiempo en segundos
A
m
p
l i t
u
d
Seales relacionadas con la tarea 1 cuando dt = 0.001
0 0.5 1 1.5 2 2.5 3 3.5 4 4.5 5
x 10
-3
10
-6
10
-5
10
-4
10
-3
10
-2
Intervalo de muestreo en segundos
E
r
r
o
r

c
u
a
d
r
a
d
o

m
e
d
i
o
MSE de la aproximacin en el intervalo [0, 0.08]
Universidad Distrital Francisco Jos de Caldas - Anlisis de Seales y Sistemas - Marco A. Alzate 29
Antes de terminar esta clase, es interesante hacer notar que el xito de Matlab como herramienta de
computacin cientfica para ingeniera se debe, fundamentalmente, a su capacidad de trabajar
fcilmente con matrices y vectores, que es el lenguaje natural de la ingeniera. Efectivamente, por
un lado ste es el tipo de clculos ms comunes en ingeniera; pero, por otro lado, y ms importante
an, los principales modelos matemticos en ingeniera hacen referencia a vectores abstractos en
espacios vectoriales abstractos, como tendremos oportunidad de ver ms adelante en el caso de las
seales y los sistemas. Esto implica un cambio de mentalidad respecto a la programacin tpica de
computadores con otros lenguajes de propsito general, en los que se necesitan extensos lazos for-
next para hacer operaciones sencillas entre vectores, por ejemplo. Como ilustracin, ntese que
para calcular la energa de una seal de duracin finita en tiempo discreto basta con encontrar el
producto interno de la seal con ella misma:

>> x = randn(100,1);
>> E = x'*x;

pero un estudiante con mentalidad escalar no dudara en cambiar la segunda lnea por una iteracin
for/next :

>> E = 0;
>> for i=1:length(x)
E = E + x(i)*x(i);
end

lo cual implica una gran ineficiencia en comparacin con la primera propuesta. Pero, como dijimos,
no es slo un asunto de eficiencia computacional sino, principalmente, un asunto de mentalidad
vectorial. Por ejemplo, cuando ve una ecuacin como la siguiente:
10
0
[ ] [ ], 0,...,10
k
k
y n a x n k n
=
= =


un estudiante con mentalidad escalar pensar en la siguiente implementacin:

for n=0:10
y(n+1)=0;
for k=0:10
if ((n-k>=0) && (n-k<=10))
y(n+1) = y(n+1) + a(k+1)*x(n-k+1);
end
end
end

con lo cual demuestra que interpreta la ecuacin anterior como una definicin para cada una de las
11 muestras de la seal y[], lo cual no est mal. Sin embargo, un estudiante con mentalidad
vectorial sabe que la ecuacin anterior tambin se puede interpretar como si la seal completa {y[n],
n=0,,10} fuera una combinacin lineal de las once seales {x[n-k], n=0,,10}
k=0,,10
,
{ } { }
10
0
[ ], 0,...,10 [ ], 0,...,10
k
k
y n n a x n k n
=
= = =


por lo que su implementacin sera as:

Universidad Distrital Francisco Jos de Caldas - Anlisis de Seales y Sistemas - Marco A. Alzate 30
y = zeros(11,1);
for k=0:10
y = y + a(k+1)*x;
x = [0; x(1:10)];
end

Otro estudiante ligeramente ms brillante notar que la ecuacin anterior es una sola ecuacin
matricial
| | | | | |
11 1 11 11 11 1
y X a

=

donde [y] es un vector columna con las muestras de la seal y[], [a] es un vector columna con los
coeficientes {a
k
}, y [X] es una matriz 1111 obtenida de la seal x[]. Tal vez este estudiante
preferira la siguiente implementacin, aunque ocupe ms memoria:

X = zeros(11,11);
X(:,1) = x;
for k=1:10
X(:,k+1) = [0; X(1:10,k)];
end
y = X*a;

Es importante notar que las tres expresiones anteriores son fundamentalmente equivalentes:

10
0
[ ] [ ], 0,...,10
k
k
y n a x n k n
=
= =


{ } { }
10
0
[ ], 0,...,10 [ ], 0,...,10
k
k
y n n a x n k n
=
= = =


| | | | | |
11 1 11 11 11 1
y X a

=

Sin embargo, la primera enfatiza una forma de calcular cada uno de los componentes de la seal
y[], la segunda enfatiza una relacin vectorial en la que el vector de correspondiente a la seal y[]
es una combinacin lineal de los vectores correspondientes a los desplazamientos de la seal x[], y
la tercera es una representacin matricial de la transformacin lineal que implica dicha
combinacin. La naturaleza matricial de Matlab enfatiza las dos ltimas interpretaciones, lo cual
ofrece muchas ventajas con respecto a la comprensin misma de la teora de seales. Con la
prctica, el estudiante alcanzar rpidamente la mentalidad vectorial y matricial que se requiere para
sacar el mximo provecho de Matlab y el mximo provecho de este curso.

Como conclusin de esta clase, nos hemos empezado a familiarizar con el programa de
computacin cientfica y visualizacin de datos ms utilizado en docencia e investigacin en
ingeniera. La interactividad de Matlab y el muy alto nivel de sus instrucciones le permiten al
usuario probar y depurar los programas enfocndose en los principios cientficos que su programa
evala y no en los detalles particulares de la programacin misma (no se deben declarar los tipos de
las variables ni los tamaos de los arreglos, se pueden construir complejas grficas con unas pocas
instrucciones, etc.). Por ltimo, su orientacin matricial no slo simplifica la programacin, sino
que facilita un adecuado modelamiento matemtico del problema que se trata.





Universidad Distrital Francisco Jos de Caldas - Anlisis de Seales y Sistemas - Marco A. Alzate 31
clear all
x=[1 2 3 4 5 6 5 4 3 2 1]';
a=[1 2 -1 2 1 -2 1 2 -1 2 1]';
for n=0:10
y(n+1)=0;
for k=0:10
if ((n-k>=0) && (n-k<=10))
y(n+1) = y(n+1) + a(k+1)*x(n-k+1);
end
end
end
y = y'
%%%%%%%%%%%%%%%%%%%%%%%%%%%%%%%%%%%
clear all
x=[1 2 3 4 5 6 5 4 3 2 1]';
a=[1 2 -1 2 1 -2 1 2 -1 2 1]';
y = zeros(11,1);
for k=0:10
y = y + a(k+1)*x;
x = [0; x(1:10)];
end
y = y
%%%%%%%%%%%%%%%%%%%%%%%%%%%%%%%%%%%
clear all
x=[1 2 3 4 5 6 5 4 3 2 1]';
a=[1 2 -1 2 1 -2 1 2 -1 2 1]';
X = zeros(11,11);
X(:,1) = x;
for k=1:10
X(:,k+1) = [0; X(1:10,k)];
end
y = X*a
%%%%%%%%%%%%%%%%%%%%%%%%%%%%%%%%%%%



















Universidad Distrital Francisco Jos de Caldas - Anlisis de Seales y Sistemas - Marco A. Alzate 32
Sexta clase. Transformaciones afines del tiempo. Periodicidad y simetra.

Otra clase importante de transformaciones que puede sufrir una seal es la de aquellas en que se
modifica la variable independiente (a la que hemos llamado, de manera genrica, tiempo). En
efecto, por un lado este tipo de transformaciones nos permitir definir propiedades de las seales
como la periodicidad o la simetra y, por otro lado, nos permitirn definir bloques fundamentales de
procesamiento como los retardadores o los sub-muestreadores.

Considere por ejemplo la seal x(t) mostrada en la siguiente figura junto con algunas
transformaciones afines de su variable tiempo. Las primeras dos transformaciones, de x(t) a x(t+s),
representan desplazamientos en el tiempo. Si s es una cantidad positiva, la seal se adelanta s
segundos pero, si s es una cantidad negativa, la seal se atrasa s segundos. Si observamos, por
ejemplo, la seal de eco-localizacin del murcilago, notamos que el odo del murcilago percibe la
superposicin de la seal transmitida con una copia retardada de la misma, la cual le indica la
distancia y la direccin del objeto que provoc la reflexin de la seal.

Figura 24. Transformaciones del tiempo continuo

La tercera transformacin muestra una inversin en el tiempo, en la que la seal se refleja con
respecto al eje t=0. Un ejemplo ilustrador sera la reproduccin de una cinta de audio al revs (en la
direccin opuesta a aquella con la que se grab). Las siguientes dos transformaciones se refieren a
un cambio de la escala de tiempo, de x(t) a x(at). Si a es mayor que uno la seal se contrae, mientras
que si a es menor que uno la seal se estira en el tiempo. Este efecto se consigue, en el ejemplo
anterior, reproduciendo la cinta de audio a una velocidad diferente a aquella con la que se grab
(ms rpidamente si a>1 y ms lentamente si a<1). Estas transformaciones se pueden combinar,
como en las ltimas tres transformaciones de la figura anterior.

Por supuesto, este tipo de transformaciones sobre la variable independiente se puede realizar
tambin en seales de tiempo discreto, aunque en este caso las operaciones de escalamiento se
-10 -5 0 5 10
0
0.5
1
x(t)
-10 -5 0 5 10
0
0.5
1
x(t+3)
-10 -5 0 5 10
0
0.5
1
x(t-3)
-10 -5 0 5 10
0
0.5
1
x(-t)
-10 -5 0 5 10
0
0.5
1
x(2t)
-10 -5 0 5 10
0
0.5
1
x(t/2)
-10 -5 0 5 10
0
0.5
1
x(3-t)
-10 -5 0 5 10
0
0.5
1
x(-3-t)
-10 -5 0 5 10
0
0.5
1
x(6-2t)
Universidad Distrital Francisco Jos de Caldas - Anlisis de Seales y Sistemas - Marco A. Alzate 33
deben considerar con ms cuidado. Como acabamos de ver, cuando se cambia de escala en el
tiempo continuo, la forma de la seal permanece intacta y completa, slo que se estira o se
comprime segn el cambio de escala. Pero, cuando trabajamos en tiempo discreto, las cosas son
diferentes. En efecto, al calcular x[2n] estamos eliminando una de cada dos muestras, cuando cada
una de ellas puede llevar informacin muy importante. Y al calcular x[n/2], aunque no estamos
perdiendo informacin, s debemos inventar nuevas muestras que correspondan a los puntos
intermedios entre las muestras originales de x[n]. En efecto, al hacer x
+2
[n] = x[2n] slo estamos
adquiriendo las muestras pares de x[n]. Y al hacer x
|2
[n] = x[n/2], estamos colocando las muestras
originales de x[n] en las muestras pares de x
|2
[n], por lo que debemos rellenar las muestras impares
con algn otro valor (lo acostumbrado es rellenar con ceros para interpolar despus mediante un
proceso de filtrado).

Figura 25. Transformaciones del tiempo discreto

Como ejercicio en clase, pedirles que encuentren x(23t/2) y x[32n] cuando x(t) y x[n] son como
se muestra en la siguiente figura.

Figura 26. Ejercicio en clase

Las transformaciones que hemos hecho de la variable tiempo nos permiten hacer dos tipos de
clasificacin de seales que sern de gran importancia en el tratamiento que hagamos de las seales
ms adelante: Seales peridicas o aperidicas, por un lado, y seales pares, impares o asimtricas,
por otro.

Una seal en tiempo continuo, x(t), es peridica con perodo Te
+
si x(t) = x(t+T) te.
Similarmente, una seal en tiempo discreto, x[n], es peridica con perodo Ne si x[n] = x[n+N]
-40 -30 -20 -10 0 10 20 30 40
0
0.5
1
x[n]
-40 -30 -20 -10 0 10 20 30 40
0
0.5
1
x[n+10]
-40 -30 -20 -10 0 10 20 30 40
0
0.5
1
x[n-10]
-40 -30 -20 -10 0 10 20 30 40
0
0.5
1
x[-n]
-40 -30 -20 -10 0 10 20 30 40
0
0.5
1
x[2n]
-40 -30 -20 -10 0 10 20 30 40
0
0.5
1
x[n/2]
-2 -1 0 1 2 3 4
0
0.2
0.4
0.6
0.8
1
-8 -6 -4 -2 0 2 4 6 8
0
0.2
0.4
0.6
0.8
1
Universidad Distrital Francisco Jos de Caldas - Anlisis de Seales y Sistemas - Marco A. Alzate 34
ne. Ntese que, si aplicamos recursivamente la definicin, tambin se cumple que x(t) = x(tkT)
te, ke y que x[n] = x[nkN] ne, ke. Por eso, se le llama perodo fundamental de la seal
al mnimo valor de T>0 de N>0 que satisface la definicin.

Figura 27. Seales peridicas en tiempo continuo y en tiempo discreto

Una seal en tiempo continuo, x(t), es simtrica con simetra par si x(t) = x(t) te. Una seal en
tiempo continuo, x(t), es simtrica con simetra impar si x(t) = x(t) te. En otro caso, la seal
es asimtrica. Aunque las anteriores definiciones de simetra se consideran con respecto al origen
del tiempo, tambin se pueden considerar seales simtricas a aquellas que tienen simetra con
respecto a otros instantes de tiempo (x(t
0
+t) = x(t
0
t) te en el caso par, o x(t
0
+t) = x(t
0
t) te
en el caso impar). En el tiempo discreto aplican idnticas definiciones: Una seal en tiempo
discreto, x[n], es simtrica con simetra par si x[n] = x[n] ne. Una seal en tiempo discreto,
x[n], es simtrica con simetra impar si x(t) = x[n] ne. En otro caso, la seal es asimtrica. La
simetra en tiempo discreto tambin se puede considerar con respecto a algn instante de tiempo
diferente al origen (x[n
0
+n] = x[n
0
n] ne en el caso par, o x[n
0
+n] = x[n
0
n] ne en el caso
impar).
Asimtrica en tiempo continuo Simtrica con simetra par en
tiempo continuo
Simtrica con simetra impar en
tiempo continuo


Asimtrica en tiempo discreto Simtrica con simetra par en
tiempo discreto
Simtrica con simetra impar en
tiempo discreto



Figura 28. Seales simtricas y asimtricas en tiempo continuo y discreto

-1 -0.5 0 0.5 1
-3
-2.5
-2
-1.5
-1
-0.5
0
0.5
1
1.5
2
Seal peridica en tiempo continuo con perodo 1/3
-20 -10 0 10 20
-3
-2.5
-2
-1.5
-1
-0.5
0
0.5
1
1.5
2
Seal peridica en tiempo discreto con perodo 5
-1 -0.8 -0.6 -0.4 -0.2 0 0.2 0.4 0.6 0.8 1
-0.4
-0.3
-0.2
-0.1
0
0.1
0.2
0.3
0.4
0.5
0.6
-1 -0.8 -0.6 -0.4 -0.2 0 0.2 0.4 0.6 0.8 1
0.1
0.2
0.3
0.4
0.5
0.6
0.7
0.8
0.9
1
-1 -0.8 -0.6 -0.4 -0.2 0 0.2 0.4 0.6 0.8 1
-0.3
-0.2
-0.1
0
0.1
0.2
0.3
-10 -8 -6 -4 -2 0 2 4 6 8 10
-0.4
-0.3
-0.2
-0.1
0
0.1
0.2
0.3
0.4
0.5
0.6
-10 -8 -6 -4 -2 0 2 4 6 8 10
0
0.1
0.2
0.3
0.4
0.5
0.6
0.7
0.8
0.9
1
-10 -8 -6 -4 -2 0 2 4 6 8 10
-0.3
-0.2
-0.1
0
0.1
0.2
0.3
Universidad Distrital Francisco Jos de Caldas - Anlisis de Seales y Sistemas - Marco A. Alzate 35
Ntese que para cualquier seal simtrica con simetra impar es necesario que x(0) = 0 (o que x[0] =
0 en el caso de tiempo discreto). En efecto, por definicin, x(0) = x(0), lo cual slo se puede
cumplir si x(0)=0.

Hay un hecho muy til con respecto a la simetra: cualquier seal se puede representar como la
suma de una seal par y una seal impar. En efecto, a cualquier seal en tiempo continuo x(t), le
podemos asociar una seal par x
e
(t) =(x(t) + x(t))/2 y una seal impar x
o
(t) =(x(t) x(t))/2.
Claramente, x(t) = x
e
(t) + x
o
(t). La misma descomposicin aplica para seales en tiempo discreto,
como muestra la siguiente figura.
Seal asimtrica x[n] Componente par, x
e
[n] Componente impar, x
o
[n]



Figura 29. Descomposicin de una seal en sus partes par e impar

Como ejercicio en clase, pedirles que encuentren las componentes par e impar de las seales usadas
en el anterior ejercicio en clase.

Antes de pasar a ver algunas seales particulares de importancia, sera interesante notar lo siguiente:
Suponga una seal real en tiempo discreto de duracin finita, con 2N1 muestras para algn Ne,
{x[n], n=1N, 2N,, 1, 0, 1,, N2, N1} (para cualquier otro valor de n con n >N, x[n]=0).
Por ser de duracin finita, si sus amplitudes son acotadas, se trata de una seal de energa.
Expresando la seal en trminos de sus componentes par e impar, la energa total de {x[n]} es
( )
1 1 1 1 1
2
2 2 2
1 1 1 1 1
[ ] [ ] [ ] [ ] 2 [ ] [ ] [ ]
N N N N N
x e o e e o o
n N n N n N n N n N
E x n x n x n x n x n x n x n

= = = = =
= = + = + +


El producto x
e
[n]x
o
[n] es una seal impar puesto que x
e
[n]x
o
[n] = (x
e
[n])(x
o
[n]) = x
e
[n]x
o
[n], por
lo que la suma de sus muestras es cero, como verificamos enseguida:
1 1
1 1
[ ] [ ] [ ] [ ] [0] [0]
N
e o e o e o
n N n N
x n x n x n x n x x

= =
= +

( ) ( )
1
1
1 1
1 1
[ ] [ ]
[ ] [ ] [ ] [ ] [ ] [ ] [ ] [ ]
0
N
e o
n
N N
e o e o e o e o
n n
x n x n
x n x n x n x n x n x n x n x n

=

= =
+
= + =
=



En consecuencia, la energa total de la seal es la suma de las energas de su componente par y su
componente impar:
1 1 1
2 2 2
1 1 1
[ ] [ ] [ ]
N N N
x e o xe xo
n N n N n N
E x n x n x n E E

= = =
= = + = +


Este resultado, aparentemente trivial, tendr un significado fundamental para nosotros. Si pensamos
que cada una de las seales x[n], x
e
[n] y x
o
[n] son vectores de un espacio vectorial (2N1)-
dimensional, donde cada muestra es un componente perpendicular, el cuadrado de la norma del
-10 -8 -6 -4 -2 0 2 4 6 8 10
-0.6
-0.4
-0.2
0
0.2
0.4
0.6
0.8
-10 -8 -6 -4 -2 0 2 4 6 8 10
-0.25
-0.2
-0.15
-0.1
-0.05
0
0.05
0.1
0.15
0.2
-10 -8 -6 -4 -2 0 2 4 6 8 10
-0.5
-0.4
-0.3
-0.2
-0.1
0
0.1
0.2
0.3
0.4
0.5
Universidad Distrital Francisco Jos de Caldas - Anlisis de Seales y Sistemas - Marco A. Alzate 36
vector sera la energa de la seal, lo cual se asocia perfectamente bien con el producto interno entre
vectores. Por ejemplo, en el caso N=2, la seal tiene tres componentes {, 0, x[-1], x[0], x[1], 0,
}, lo que se puede interpretar como la suma de tres seales perpendiculares {, 0, x[-1], 0, 0, 0,
}, {, 0, 0, x[0], 0, 0, } y {, 0, 0, 0, x[1], 0, } (el producto interno entre dos de ellas es
cero). De esta manera, la definicin de energa sera slo una expresin del teorema de Pitgoras,
como muestra la siguiente figura, pues la norma euclidiana del vector es la raz cuadrada de su
energa.

Figura 30. Interpretacin de las seales de energa como vectores en un espacio vectorial abstracto

De acuerdo con esta interpretacin, las seales de energa forman un espacio vectorial en el que
cada vector (cada seal) se puede expresar como la combinacin de dos vectores (seales) que
pertenecen a sub-espacios ortogonales entre ellos: el de las seales pares y el de las seales impares.
Estos dos sub-espacios son complementarios en el sentido en que su suma forma el espacio total de
las seales de energa. Ms adelante enfatizaremos esta interpretacin, de manera que podremos
notar, por ejemplo, que la transformada de Fourier es solamente un cambio de base para expresar el
espacio vectorial de las seales de energa, o que la aproximacin de una seal mediante otra
tratando de minimizar el error cuadrado promedio, es solamente una proyeccin perpendicular de
un vector sobre una sub-espacio vectorial.

Figura 31. Las seales pares e impares de energa forman sub-espacios complementarios ortogonales del espacio
vectorial de las seales de energa


x[0]
x[-1]
x[1]
n = 0
n = -1
n = 1
Espacio vectorial de las seales
de energa con simetra par
Espacio vectorial de las seales
de energa con simetra impar
Espacio vectorial de todas
las seales de energa
Universidad Distrital Francisco Jos de Caldas - Anlisis de Seales y Sistemas - Marco A. Alzate 37
Segunda Tarea

1. Muestre que si x[n] es una seal peridica, su potencia se puede encontrar promediando la
potencia instantnea en un perodo.

2. Determine si las siguientes seales son de energa, de potencia o ninguna de ellas:
(a)
1 0
( )
0 0
t
u t
t
>
=

<


(b)
1 0
[ ]
0 0
n
u n
n
>
=

<


(c) ( ) ( ), 0
at
x t e u t a

= >
(d) ( ) ( ) x t tu t =
(e) [ ] ( 0.5) [ ]
n
x n u n =
(f)
3
[ ] 2
j n
x n e =

3. Muestre que la representacin de una seal como suma de una componente par y otra
componente impar es nica.

4. Muestre que el producto de dos seales pares o impares es una seal par y que el producto de
una seal par y otra impar es una seal impar.

5. Para la seal x(t) mostrada a la izquierda en la siguiente figura, dibuje x(1 2t/3). Para la seal
x[n] mostrada a la derecha en la siguiente figura, dibuje x[1 2n/3].



6. Dibuje (etiquetando bien sus dibujos) las componentes par e impar de las siguientes seales.

-2 -1 0 1 2 3 4 5
0
0.5
1
1.5
2
2.5
3
-5 0 5 10 15 20
0
0.5
1
1.5
2
2.5
3
-3 -2 -1 0 1 2 3 4 5 6
0
0.5
1
-4 -3 -2 -1 0 1 2
0
0.5
1
-3 -2 -1 0 1 2 3 4 5
0
0.5
1
1.5
2
-2 -1 0 1 2 3 4 5
0
0.5
1
1.5
2
Universidad Distrital Francisco Jos de Caldas - Anlisis de Seales y Sistemas - Marco A. Alzate 38
Sptima clase. Seales exponenciales. Periodicidad en tiempo continuo y en tiempo discreto.

(Discusin de la segunda tarea)

En la naturaleza son comunes los sistemas caracterizados por seales cuyas tasas de crecimiento son
proporcionales a las seales mismas. Por ejemplo, en ausencia de predadores y abundancia de
recursos, la tasa de crecimiento de la poblacin de una especie es proporcional al nmero de
individuos,
( ) ( )
d
x t x t
dt
o =
Considerando o=1, estamos ante una funcin que es idntica a su derivada Cul es esa funcin?
Si aplicamos el mtodo de la serie de potencias,
0
( )
n
n
n
x t a t

=
=

, obtenemos
( )
1 0 0 0 0
1 1 2 3
1 0
1 , , ,...,
1 1 2 1 2 3 !
n n
n n n n n
n n
a a a a
na t a t n a a a a a a
n

+
= =
= + = = = = =



Como queremos conservar la misma tasa de crecimiento en t=0, necesitamos x(0)=a
0
=1, de donde
obtenemos la funcin que es idntica a su derivada (nica, excepto por un factor constante):
0
( ) ( ) ( )
!
n
n
d t
x t x t x t
dt n

=
= =


Por otro lado, ntese que
0 0
1
lim lim
t t
t t
d r r r
r r
dt
+A A
A A
| |
= =
|
A A
\ .

de manera que, si
0
1
lim 1
r
A
A
| |
=
|
A
\ .
, r
t
es idntica a su derivada, esto es,
0
!
n
t
n
t
r
n

=
=

. Evaluando en
t=1, obtenemos el valor de r que satisface dicha relacin. A ese nmero le llamamos e:
0
1
2, 71828182845904523536028747135266249775724709369995...
!
n
e
n

=
= =


El valor particular de e no es tan importante como la definicin misma de la funcin e
t
, pues gracias
a ella podemos calcular exponenciales donde el exponente es un nmero complejo, o donde el
exponente es una matriz:
( )
0 0
1
,
! !
a b n
c d j
n n
n
a b j
e e
c d n n
u
u
(

(

= =
(
= =
(




Esta ltima observacin nos trae a otro nmero interesante, j, que se define mediante la relacin
j
2
=-1 de manera que, por ejemplo, \(-25) = \[(25)(-1)]=\(25)\(-1)=5j. A los nmeros que tienen a j
como factor les decimos imaginarios, pero no es "uno" tan imaginario como j? Despus de todo, el
concepto mismo de nmero es una abstraccin terica de algo ms real: la cardinalidad de los
conjuntos. Podemos saber qu son tres vacas o qu son cinco asientos, pero qu es el "tres" qu
es el "cinco"? Sin embargo, como el proceso de contar se refiere a establecer una relacin biunvoca
entre los elementos de un conjunto y los nmeros naturales, no parece apropiado decir que los
Universidad Distrital Francisco Jos de Caldas - Anlisis de Seales y Sistemas - Marco A. Alzate 39
nmeros naturales sean imaginarios, pero lo cierto es que son el resultado de la imaginacin del ser
humano. De hecho, cuando se introdujo el nmero cero fue necesario hacer una mayor abstraccin,
un mayor esfuerzo de la imaginacin. Y mayor esfuerzo an se requiri al introducir los nmeros
negativos. En efecto, dentro del sistema de los naturales no es posible encontrar un nmero que
sumado a cinco d tres, pero se puede extender el sistema a los nmeros enteros para "darle
existencia" a tal nmero, al cual llamamos "menos dos", aunque ya no quede fcil hablar de menos
dos vacas o de menos dos asientos. De igual manera podemos ir extendiendo los sistemas
numricos: Cul es el nmero que, al multiplicarlo por dos, da tres? Ese nmero no existe entre los
naturales y tampoco existe entre los enteros, pero podemos inventar (imaginar) un sistema
extendido que lo incluya: los nmeros racionales. Y cul es el nmero que multiplicado por si
mismo da dos? Ese nmero no existe entre los naturales, ni entre los enteros, ni entre los racionales,
pero podemos inventar (imaginar) un sistema extendido que lo incluya: los nmeros reales. De los
naturales a los enteros, de los enteros a los racionales, de los racionales a los reales, slo hemos
extendido la imaginacin: Todos son nmeros "imaginarios" cuyas propiedades se asocian con el
concepto primigenio de "contar". Ahora preguntamos Cul es el nmero que multiplicado por s
mismo da "menos uno"? Ese nmero no existe en los naturales, ni en los enteros, ni en los
racionales, ni en los reales, por lo que inventamos otro sistema de numeracin: Los imaginarios
qu son tan reales como los dems! O, mejor dicho, que son tan imaginarios como los dems. Por
ltimo, si nos preguntamos cules son las soluciones de la ecuacin x
2
+ x + 1 = 0, notaremos que
esos nmeros no estn en los naturales, los enteros, los racionales, los reales o los imaginarios: Por
eso imaginamos el sistema de numeracin de los complejos. Como de costumbre, de la misma
manera que la abstraccin terica de las seales como funciones matemticas nos permite modelar
el concepto de seal (magnitud fsica medible que cambia con el tiempo), la abstraccin terica de
los sistemas de numeracin representa conceptos muy reales y muy concretos. En particular, el
nmero irracional e y el nmero imaginario j juegan un papel fundamental en el modelamiento de
las seales, como mencionaremos en breve.

Con esta introduccin a los nmeros e y j, y habiendo definido seales, sistemas, potencia, energa,
simetra y periodicidad, ahora revisaremos varias seales bsicas en tiempo continuo y en tiempo
discreto que nos servirn para construir muchas otras seales.

La primera de ellas es la seal exponencial, ( ) ,
t
x t Ae t
o
= e , donde A y o pueden ser, en general,
nmeros complejos. Consideremos primero el caso en que ambos parmetros son reales. En el
origen la seal toma el valor x(0) = A. En otros instantes de tiempo, el comportamiento de la seal
depende del signo del parmetro o : x(t) crece con t si o es positiva y decrece con t si o es
negativa, como se muestra en la siguiente figura. Por supuesto, si o es igual a cero, la seal es una
constante x(t)=A te.
Universidad Distrital Francisco Jos de Caldas - Anlisis de Seales y Sistemas - Marco A. Alzate 40

Figura 32. Seales exponenciales reales en tiempo continuo

Un caso ms interesante para estudiar en este curso es cuando o es un nmero puramente
imaginario, o=je, esto es, cuando ( ) ,
j t
x t Ae t
e
= e , donde A sigue siendo real. Una forma
tradicional de representar est seal es como un punto sobre el crculo de radio A en el plano
complejo, que gira a una velocidad de e radianes por segundo. Esta interpretacin cobra sentido
cuando consideramos la relacin de Euler:
( )
( ) ( )
2 2 1
0 0 0
1 1
! (2 )! (2 1)!
n
n n
n n
j
n n n
j
e j
n n n
u
u u u
+
= = =
( (
= = +
( (
+



cos( ) sin( )
j
e j
u
u u = +

Esto es, si interpretamos e
ju
como un punto en el plano complejo sobre el crculo unitario que forma
un ngulo u con el eje horizontal, notamos que su componente real es la proyeccin del punto sobre
el eje horizontal, cos(u), y su componente imaginaria es la proyeccin del punto sobre el eje
vertical, sin(u), como muestra la figura 33. Esta representacin, conocida como el diagrama de
Argand, explica la conversin entre las formas polar y rectangular de los nmeros complejos:
( ) ( )
2 2 1
, tan
cos , sin
j
b
Me a jb M a b
a
a M b M
u
u
u u

| |
= + = + =
|
\ .
= =


x(t)=Ae
ot
, A>0, o>0
A
t
x(t)=Ae
ot
, A>0, o<0
A
t
x(t)=Ae
ot
, A<0, o>0
A
t
x(t)=Ae
ot
, A<0, o<0
A
t
Universidad Distrital Francisco Jos de Caldas - Anlisis de Seales y Sistemas - Marco A. Alzate 41

Figura 33. Partes real e imaginaria de la exponencial compleja

Ntese en la figura anterior cun fcil es interpretar una frecuencia negativa: simplemente ocurre
cuando el punto en el plano complejo da vueltas en el sentido de las manecillas del reloj, pues las
frecuencias positivas lo hacen girar en el sentido contrario a las manecillas del reloj. De aqu la
simetra par de la funcin coseno y la simetra impar de la funcin seno. En efecto, la relacin de
Euler y su conjugado complejo toman las siguientes formas:
e
jet
= cos(et) + jsin(et) e
-jet
= cos(et) jsin(et)
Sumando y restando los dos trminos anteriores,
( ) ( )
1 1
cos( ) sin( )
2 2
j t j t j t j t
t e e t e e
j
e e e e
e e

= + =

Estas expresiones se pueden interpretar como la suma (y diferencia) de dos puntos sobre el crculo
unitario que se mueven en direcciones opuestas.

Figura 34. La suma de dos puntos del plano complejo que se mueven en direcciones opuestas sobre el crculo
unitario siempre cae sobre el eje real. La diferencia siempre cae sobre el eje imaginario.

(para los matemticos, las anteriores expresiones de seno y coseno son las definiciones mismas de
dichas funciones, donde e
ju
es slo una serie de potencias: nada de catetos ni de hipotenusas!)
et
1
et
2
et
3
e
t
1
e
t
2
e
t
3
cos( ) sin( )
j t
Ae A t jA t
e
e e = +
j t
e
e
j t
e
e
j t
e
e

sin( )
2
j t j t
e e
j t
e e
e

=
cos( )
2
j t j t
e e
t
e e
e

+
=
Universidad Distrital Francisco Jos de Caldas - Anlisis de Seales y Sistemas - Marco A. Alzate 42
La relacin de Euler, e
ju
= cos(u) + j sin(u), es una de las relaciones ms celebradas en las
matemticas. Richard Feynman (1918-1988), uno de los ms grandes fsicos del siglo XX, le deca
a esta relacin la joya de las matemticas (The Feynman Lectures on Physics, vol. I, 1977). De
hecho, evalundola en u = t se obtiene una relacin muy simple entre los cinco nmeros ms
importantes de las matemticas:

Figura 35. Relacin simple entre los cinco nmeros ms importantes de las matemticas

Esta simple relacin evoca un concepto esttico: belleza. Si, como vimos, los sistemas de
numeracin son un producto de la creatividad humana, toda la matemtica lo es, tanto como la
poesa o la pintura, donde es ms fcil experimentar la belleza: Esa sensacin de profundo placer
que surge de una experiencia sensorial, en la que un maravilloso misterio por descubrir se revela en
algo tan simple como una imagen o un sonido. Siendo as, la relacin e
jt
+1=0 no es ms que un
hermoso poema.

Volviendo a nuestra seal ( )
j t
x t e
e
= , la principal propiedad de la exponencial compleja es su
periodicidad. En efecto, recordando la definicin de periodicidad ( ( ) ( ) x t T x t t + = e ),
( )
2
1, lo cual ocurre si para cualquier
j t T j t j t j T j T
k
e e e e t e T k
e e e e e
t
e
+
= = e = = e
, en cuyo
caso e
je(t+T)
ocupa la misma posicin de e
jet
en el crculo unitario, pero le ha dado k vueltas ms. En
consecuencia, el perodo fundamental de la exponencial compleja con frecuencia angular e es T
0
=
2t/e. Obsrvese que para cualquier ee, e=0, existe un T
0
(=2t/e)e y, si e=0, cualquier valor
real de T es un perodo vlido de la seal. Entonces, aunque parezca extrao hacer notar lo
siguiente, la exponencial compleja en tiempo continuo siempre es peridica. Cuando veamos en un
momento la exponencial compleja en tiempo discreto, se notar la relevancia de esta extraa
observacin.

Si la variable t se refiere al tiempo medido en segundos, e es la velocidad angular en radianes por
segundos. Como cada 2t radianes se completa un ciclo, e se puede expresar como 2tF, donde F es
la frecuencia en ciclos por segundo. El perodo, entonces, T = 2t/e = 1/F, es el tiempo que toma un
ciclo de la seal, en segundos.

Ntese que, tratndose de una seal peridica, la energa de la exponencial compleja debe ser
infinita. Pero es fcil ver que se trata de una seal de potencia:
2
1 1 2
lim lim 1 lim 1
2 2 2
j t
x
P e dt dt
t t
e
t t t t t
t
t t t

= = = =
} }


Finalmente, consideremos el caso en que o es una cantidad compleja, o = o + je, mientras A sigue
siendo un nmero real:
( )
( )
( )
t j t t j t
x t Ae Ae Ae e
o o e o e +
= = =

Universidad Distrital Francisco Jos de Caldas - Anlisis de Seales y Sistemas - Marco A. Alzate 43
Se puede notar que se trata del producto entre los dos casos anteriores: El primer trmino crece o
decrece exponencialmente de acuerdo con el signo de o, y el segundo trmino oscila a una
velocidad de e radianes por segundo. En consecuencia, de acuerdo con la relacin de Euler, las
partes real e imaginaria de x(t) se comportan como en la siguiente figura:

Figura 36. Caso ms general de la exponencial compleja

Habiendo estudiado las seales exponenciales en tiempo continuo, se debe decir que con las
exponenciales complejas en tiempo discreto todo es muy parecido, aunque existen dos diferencias
fundamentales.

Sea x[n]=Ae
on
. Nuevamente, cuando A y o son reales, tenemos los mismos casos que en el tiempo
continuo:


{ } Re ( ) , 0 x t o < { } Re ( ) , 0 x t o >
t
t
x[n]=Ae
on
, A>0, o>0
A
t
x[n]=Ae
on
, A>0, o<0
A
t
x[n]=Ae
on
, A<0, o>0
A
t
x[n]=Ae
on
, A<0, o<0
A
t
Universidad Distrital Francisco Jos de Caldas - Anlisis de Seales y Sistemas - Marco A. Alzate 44

Figura 37. Seales exponenciales reales en tiempo discreto

La figura anterior incluye una posible generalizacin que usa un escalar cualquiera r en vez de e
o
,
de manera que la seal tendra la forma x[n]=Ar
n
. En este caso, cuando r < 0, tendramos las dos
formas adicionales mostradas en la parte inferior de dicha figura.

Podemos ahora verificar la periodicidad de la exponencial en tiempo discreto cuando o es un
nmero puramente imaginario, o=je, esto es, cuando [ ] ,
j n
x n Ae n
e
= e . Por conveniencia, y sin
perder generalidad, le asignaremos a la amplitud A el valor 1. Notemos que x[n] sigue siendo un
punto del plano complejo que da vueltas al crculo unitario. La gran diferencia es que ahora lo hace
en pasos discretos de e radianes por muestra (la velocidad angular ya no est dada en radianes por
segundo sino en radianes por muestra), lo cual hace vlido preguntarnos si volver a repetir los
pasos por donde ya haba pasado en la vuelta anterior, o en un nmero finito de vueltas. Para ver
bajo qu condiciones ocurrir algo as, revisemos nuevamente la definicin de periodicidad:
( )
2
1, lo cual ocurre si para cualquier
j n N j n j n j N j N
k
e e e e n e N k
e e e e e
t
e
+
= = e = = e

Evidentemente, para que la exponencial compleja en tiempo discreto sea peridica se hace
necesario que la velocidad angular sea un mltiplo racional de 2t:
2 , ,
k
k N
N
e t = e
Esta es una de las grandes diferencias con el caso del tiempo continuo, donde las exponenciales
complejas son siempre peridicas con perodo bien definido para cualquier valor real de la
velocidad angular (T=2t/e, ee). En el caso discreto, es necesario que la velocidad angular
tenga un forma muy particular: debe ser un mltiplo racional de 2t, e=2tk/N, en cuyo caso (si k/N
es una fraccin simplificada, esto es, si k y N son primos relativos no tienen factores en comn-) el
perodo es N, que se cumple cuando el punto le ha dado k vueltas al crculo unitario. La explicacin
es simple: Como el punto en tiempo continuo pasa por todos los puntos del crculo unitario, siempre
volver al mismo punto, de manera indefectible y en una sola vuelta. Pero como el punto en tiempo
discreto no necesariamente pasa por todos los puntos, es posible que deba dar varias vueltas antes
de volver al punto original o, ms an, puede que nunca vuelva al punto original. La siguiente
figura muestra las posiciones del punto en un perodo de 5 muestras para diferentes valores de k:
x[n]=Ar
n
, A>0, r<-1
A
t
x[n]=Ar
n
, A>0, -1< r < 0
A
t
Universidad Distrital Francisco Jos de Caldas - Anlisis de Seales y Sistemas - Marco A. Alzate 45

Figura 38. La frecuencia digital es un nmero racional: El denominador N es el perodo y el numerador k es el
nmero de vueltas al crculo unitario en un perodo. En todos los casos el nmero de ciclos por muestra es 1/N

La figura anterior nos lleva a identificar la segunda gran diferencia con la exponencial compleja en
tiempo continuo. Mientras en el tiempo continuo todas las frecuencias eran distinguibles pues entre
mayor era la velocidad angular mayor era el nmero de ciclos por segundo, en el caso del tiempo
discreto hay frecuencias que no se pueden distinguir entre ellas. Por ejemplo, ntese que la
frecuencia -1/5, en donde incrementamos 2t/5 radianes por muestra en el sentido de las manecillas
del reloj, resulta idntica a la frecuencia 4/5, en la que aumentamos 8t/5 en el sentido contrario a
las manecillas del reloj con cada muestra. Ms an, despus de 4/5, la siguiente frecuencia vlida
para que el perodo siga siendo 5 es 6/5, pero al observar el orden de los puntos que traza la seal,
notamos que son los mismo y en el mismo orden que los puntos que traza cuando la frecuencia es
1/5. Esto es fcil de apreciar si consideramos que las velocidades angulares separadas por un
nmero entero de 2t son indistinguibles:
( ) 2 2
,
j k n j n j kn j n
e e e e k n
e t e t e +
= = e
Para ver con mayor claridad las diferencias fundamentales en la periodicidad de las seales en
tiempo continuo y en tiempo discreto, veamos 11 muestras de las seales mostradas en la figura
anterior

Figura 39. Diferentes exponenciales complejas en tiempo discreto (parte real en rojo, parte imaginaria en azul)
n=0,5
n=1
n=2
n=3
n=4
1
[ ] exp 2
5
x n j n t
| |
=
|
\ .
El perodo 5 se cumple al darle
una vuelta al crculo unitario
n=0,5
n=3
n=1
n=4
n=2
2
[ ] exp 2
5
x n j n t
| |
=
|
\ .
El perodo 5 se cumple al darle
dos vueltas al crculo unitario
n=0,5
n=2
n=4
n=1
n=3
3
[ ] exp 2
5
x n j n t
| |
=
|
\ .
El perodo 5 se cumple al darle
tres vueltas al crculo unitario
n=0,5
n=4
n=3
n=2
n=1
4
[ ] exp 2
5
x n j n t
| |
=
|
\ .
El perodo 5 se cumple al darle
cuatro vueltas al crculo unitario
0 5 10
-1
0
1
f = -7/5
0 5 10
-1
0
1
f = -6/5
0 5 10
0
0.5
1
f = -5/5
0 5 10
-1
0
1
f = -4/5
0 5 10
-1
0
1
f = -3/5
0 5 10
-1
0
1
f = -2/5
0 5 10
-1
0
1
f = -1/5
0 5 10
0
0.5
1
f = 0/5
0 5 10
-1
0
1
f = 1/5
0 5 10
-1
0
1
f = 2/5
0 5 10
-1
0
1
f = 3/5
0 5 10
-1
0
1
f = 4/5
0 5 10
-0.5
0
0.5
1
f = 5/5
0 5 10
-1
0
1
f = 6/5
0 5 10
-1
0
1
f = 7/5
0 5 10
-1
0
1
f = 8/5
Universidad Distrital Francisco Jos de Caldas - Anlisis de Seales y Sistemas - Marco A. Alzate 46
Ntese que, en la figura anterior, cualquier frecuencia fuera del rango {-2/5, -1/5, 0, 1/5, 2/5}se
puede confundir con una de las que se encuentran en este rango: -7/5, 3/5 y 8/5 se confunden con -
2/5; -6/5 y 4/5 se confunden con -1/5; -5/5 y 5/5 se confunden con 0/5; -4/5 y 6/5 se confunden con
1/5; y -3/5 y 7/5 se confunden con 2/5. Esta indistinguibilidad entre diferentes frecuencias en
tiempo discreto es lo que se conoce con el nombre de Alias: Cualquier frecuencia f con f>1/2 se
puede confundir con alguna otra frecuencia f con fs1/2. Ms an, ntese que la velocidad de las
oscilaciones crece mientras f va de cero hasta ciclo/muestra pero, una vez f supera ese valor, la
velocidad de las oscilaciones empieza a decaer hasta que f llega a 1 ciclo por muestra, cuando la
seal vuelve a corresponder a un nivel dc, como cuando f vala cero. La oscilacin ms rpida
posible se consigue con f = (e=t), cuando x[n] = e
jen
= (-1)
n
, como se muestra en la siguiente
figura. Cualquier otra frecuencia produce una oscilacin ms lenta:

Figura 40. e
j2
t
fn
para diferentes valores de f (parte real en rojo, parte imaginaria en azul)

Resumiendo: Mientras en tiempo continuo las exponenciales complejas son siempre peridicas, en
tiempo discreto se necesita que la velocidad angular sea un mltiplo racional de 2t. Mientras en
tiempo continuo todas las posibles frecuencias en los reales son distinguibles, en tiempo discreto
diferentes frecuencias pueden generar las mismas seales en el tiempo (alias). Como la frecuencia
ms alta en tiempo discreto es medio perodo por muestra, se suele limitar el rango de frecuencias
vlidas en el intervalo [-, ] o, lo que es lo mismo, la velocidad angular se suele limitar al
intervalo [-t, t].















-10 -5 0 5 10
-1
-0.8
-0.6
-0.4
-0.2
0
0.2
0.4
0.6
0.8
1
f = 4/10
-10 -5 0 5 10
-1
-0.8
-0.6
-0.4
-0.2
0
0.2
0.4
0.6
0.8
1
f = 5/10
-10 -5 0 5 10
-1
-0.8
-0.6
-0.4
-0.2
0
0.2
0.4
0.6
0.8
1
f = 6/10
Universidad Distrital Francisco Jos de Caldas - Anlisis de Seales y Sistemas - Marco A. Alzate 47
Octava clase. Exponenciales complejas armnicas, impulsos, escalones

Veamos algunas propiedades interesantes de las exponenciales complejas en tiempo continuo y en
tiempo discreto. Si qued claro, los estudiantes podrn resolver algunos problemas:

quiz: Cul es el perodo de
1 2
( )
j t j t
x t e e
e e
= + ? Ntese que, en t=0, ambos trminos de la suma se
encuentran en 1, de manera que x(0)=2. Cundo volver la seal a ese punto? Claramente, cada
trmino de la suma deber haber dado un nmero entero de vueltas al crculo unitario, esto es, en el
tiempo que el primer trmino da k
1
vueltas, el segundo trmino deber haber dado k
2
vueltas. Esto
slo es posible si e
1
y e
2
son mltiplos racionales entre s. En efecto, el perodo de x(t) ser T si
( ) ( )
1 2 1 1 2 2
( ) ( )
j t T j t T j t j T j t j T
x t x t T e e e e e e t
e e e e e e + +
= + = + = + e
lo cual ocurrir solamente si se satisfacen simultneamente las siguientes dos condiciones:
e
1
T=2tk
1
y e
2
T=2tk
2
. De esta manera, en T segundos el primer trmino da k
1
vueltas y el segundo
trmino da k
2
vueltas para volverse a encontrar en x(T) = x(0), como habamos considerado
originalmente. Dividiendo la primera condicin por e
2
T obtenemos la condicin de periodicidad
para x(t): e
1
/e
2
=k
1
/k
2
. Esto es, x(t) slo ser peridica si e
1
y e
2
son mltiplos racionales entre s,
en cuyo caso T=2tk
1
/e
1
= 2tk
2
/e
2
.

Por ejemplo, si e
1
=2t(1/3) y e
2
=2t(2/5), e
1
/e
2
=5/6 y T = 6/(2/5) = 15. En efecto, despus de 15
segundos, el primer trmino ha dado 5 vueltas al crculo unitario y el segundo trmino ha dado 6
vueltas, de manera que se vuelven a encontrar en x(0) = x(15) = 2. Pero si e
1
=2t(1/4) y e
2
=2t(\2),
mientras el primer trmino da una vuelta cada cuatro segundos, el segundo trmino nunca
completar una vuelta en un nmero entero de segundos y, por consiguiente, nunca se volvern a
encontrar en el punto de origen.

quiz: Cul es el perodo de
1 2
( )
j t j t
x t e e
e e
= + ? Aqu las cosas son distintas, ya que estamos
hablando de una seal real que siempre ser peridica con perodo T = 4t/(e
2
- e
1
):
2 1 2 1 2 1 2 1
2 1 2 1 2 2 2 2
( ) 2cos 2 cos
2 2
j t j t j t j t
x t e e e e t t
e e e e e e e e
e e e e
+ +
| |
| | | |
= + = =
| | |
\ . \ .
\ .

En efecto, ya no nos interesa cundo los dos trminos de la suma se volvern a encontrar en el
punto 1+j0 del crculo unitario sino cundo volvern a estar alineados para que la magnitud de la
suma vuelva a ser dos. Independientemente de que alguna vez los dos sumandos vuelvan a pasar
por el punto de origen o no, siempre volvern a pasar uno encima de otro. De hecho, lo harn (e
2
-
e
1
)/4t veces por segundo.

A la luz de las discusiones anteriores podemos estudiar las familias de exponenciales complejas
armnicamente relacionadas:
{ }
2 2
0,1,..., 1
( ) , [ ] ,
k k
j n j t
N T
k k
k k N
t e t n e n
t t
| |
e e

= e = e
` `
) )

Universidad Distrital Francisco Jos de Caldas - Anlisis de Seales y Sistemas - Marco A. Alzate 48
En el caso del tiempo continuo, el perodo fundamental de la k-sima exponencial, |
k
(t), es T/k.
Claro, como una seal que se repite cada t segundos tambin se repite cada kt segundos para
cualquier ke, todas las seales de la familia comparten el perodo comn T. Igualmente, ntese
que para dos valores diferentes k
1
y k
2
, los perodos de las seales correspondientes son diferentes,
T/k
1
y T/k
2
y, por lo tanto, cada una de las seales de la familia es perfectamente distinguible de
todas las dems.

Figura 41. Cinco elementos de la familia exponencial en tiempo continuo con perodo 5

En el caso del tiempo discreto, el perodo fundamental de la k-sima exponencial, |
k
[n], es
N/gcd(k,N), de manera que las funciones |
k
[n] y |
kmN
[n] son indistinguibles. Por lo tanto, para
evitar fenmenos de alias, en el caso de tiempo discreto la familia de exponenciales complejas
armnicamente relacionadas se reduce a N seales bsicas, tpicamente correspondiente al rango de
k entre 0 y N-1. De todas maneras, si N/gcd(k,N) es el perodo fundamental, N es otro perodo de la
k-sima seal y, por lo tanto, todas las seales en ese rango comparten un perodo en comn N.

Figura 42. Nueve elementos de la familia exponencial en tiempo discreto con perodo 5. Ntese que k=-2 no se
distingue de k=3, k=-1 no se distingue de k=4, k=0 no se distingue de k=5 y k=1 no se distingue de k=6. Por eso,
para N=5, slo se usa el rango ke{0,1,2,3,4}
0 0.5 1 1.5 2 2.5 3 3.5 4 4.5 5
-1
-0.5
0
0.5
1
tiempo
A
m
p
l
i
t
u
d
Im[phi
k
(t)] = sin(2 pi k t / 5)


k=-2
k=-1
k=0
k=1
k=2
-10 -5 0 5 10
-1
-0.5
0
0.5
1
Im(phi
k
[n]) = sin(2 pi k n / 5), k=-2
-10 -5 0 5 10
-1
-0.5
0
0.5
1
Im(phi
k
[n]) = sin(2 pi k n / 5), k=-1
-10 -5 0 5 10
-1
-0.5
0
0.5
1
Im(phi
k
[n]) = sin(2 pi k n / 5), k=0
-10 -5 0 5 10
-1
-0.5
0
0.5
1
Im(phi
k
[n]) = sin(2 pi k n / 5), k=1
-10 -5 0 5 10
-1
-0.5
0
0.5
1
Im(phi
k
[n]) = sin(2 pi k n / 5), k=2
-10 -5 0 5 10
-1
-0.5
0
0.5
1
Im(phi
k
[n]) = sin(2 pi k n / 5), k=3
-10 -5 0 5 10
-1
-0.5
0
0.5
1
Im(phi
k
[n]) = sin(2 pi k n / 5), k=4
-10 -5 0 5 10
-1
-0.5
0
0.5
1
Im(phi
k
[n]) = sin(2 pi k n / 5), k=5
-10 -5 0 5 10
-1
-0.5
0
0.5
1
Im(phi
k
[n]) = sin(2 pi k n / 5), k=6
Universidad Distrital Francisco Jos de Caldas - Anlisis de Seales y Sistemas - Marco A. Alzate 49
Ahora bien, si todas las seales de una familia de exponenciales complejas armnicamente
relacionadas tienen un perodo en comn, cualquier combinacin lineal de las mismas ser una
seal peridica con el mismo perodo:
1
2 2
0
( ) [ ]
Seal peridica en tiempo Seal peridica en tiempo
continuo con perodo discreto con perodo
k k
N
j n j t
N T
T k N k
k k
x t a e x n b e
T N
t t

= =
= =



Una pregunta interesante por hacerse es si todas las seales peridicas tienen una representacin
semejante. La respuesta la estudiaremos con cuidado ms adelante (ver dcimo-sexta clase), pero
adelantaremos que, bajo condiciones muy generales y aceptando diferentes formas de convergencia
de seales, todas las seales peridicas se pueden representar como una combinacin lineal de
exponenciales complejas armnicamente relacionadas. En la dcima clase veremos que lo que
hemos conseguido es expandir el subespacio vectorial de las seales peridicas de perodo dado en
una base ortonormal particular, la base de las exponenciales complejas armnicamente
relacionadas. Esta es una interpretacin fundamental de las seales que iremos introduciendo
lentamente en este curso.

Existe una gran cantidad de bases (ortonormales o no) sobre las cuales podemos expandir sub-
espacios de seales ms generales. A continuacin veremos algunas de ellas.

Tal vez la seal ms simple en tiempo discreto es el impulso unitario:

Figura 43. Definicin del impulso unitario en tiempo discreto

Otra seal bsica en tiempo discreto es el escaln unitario,
-5 -4 -3 -2 -1 0 1 2 3 4 5
-0.2
0
0.2
0.4
0.6
0.8
1
Tiempo, n
A
m
p
l
i
t
u
d
o[n]
| |
1 0
0 0
n
n
n
o
=
=

=

Universidad Distrital Francisco Jos de Caldas - Anlisis de Seales y Sistemas - Marco A. Alzate 50

Figura 44. Definicin del escaln unitario en tiempo discreto

Entre estas dos seales existe una relacin muy interesante:

o[n] = u[n] u[n-1]


Figura 45. El impulso unitario es la primera diferencia del escaln unitario

Despejando u[n] e iterando sobre n,
0
[ ] [ ] [ 1] [ ] [ 1] [ 2] [ ]
k
u n n u n n n u n n k o o o o

=
= + = + + = =


Esta ltima relacin se puede interpretar como la suma de un nmero infinito de seales, como
muestra la siguiente figura.

Podemos expresar la anterior relacin entre el escaln y el impulso unitarios de la siguiente manera:
[ ] [ ] [ ],
k
u n u k n k n o

=
= e


pues, efectivamente, para valores negativos de k, u[k] es cero y, para valores no negativos de k, u[k]
es uno. Es importante notar que, aunque la anterior ecuacin se puede referir a la forma de calcular
la n-sima muestra de u[n], en realidad la variable n recorre todo el rango del tiempo discreto
mientras que la variable k es simplemente un ndice que recorre las seales que estamos
combinando y el respectivo coeficiente escalar. Preferiramos escribirla de la siguiente manera
-5 0 5
-0.2
0
0.2
0.4
0.6
0.8
1
Tiempo, n
A
m
p
l
i
t
u
d
u[n]
| |
1 0
0 0
n
u n
n
>
=

<

-5 -4 -3 -2 -1 0 1 2 3 4 5
0
0.5
1
-5 -4 -3 -2 -1 0 1 2 3 4 5
-1
-0.5
0
-5 -4 -3 -2 -1 0 1 2 3 4 5
0
0.5
1
u[n]
-u[n-1]
o[n] = u[n]-u[n-1]
Universidad Distrital Francisco Jos de Caldas - Anlisis de Seales y Sistemas - Marco A. Alzate 51
{ } { } [ ], [ ] [ ],
k
u n n u k n k n o

=
e = e


para dejar bien explcito que la seal escaln unitario es una combinacin lineal de las seales
impulsos unitarios desplazados (vase la ltima parte de la quinta clase). Es sta la interpretacin
que queremos darle a la suma anterior, que podemos extender a cualquier seal en tiempo discreto.

Figura 46. El escaln unitario es la suma de impulsos unitarios desplazados k unidades de tiempo, para k=0,1,2,

Considrese cualquier seal {x[n], ne} y su producto con el impulso unitario, {o[n], ne}.
Cuando n=0, o[n] es cero y, por consiguiente, x[n]o[n] tambin es cero. Pero, para n=0, o[n] es uno
y, por consiguiente, x[n]o[n] es x[0]. En consecuencia, la seal {x[n]o[n], ne} es idntica a x[0]
veces la seal {o[n], ne}:
{ } { } [ ] [ ], [0] [ ], x n n n x n n o o e = e
o, en trminos escalares,
[ ] [ ] [0] [ ], x n n x n n o o = e
De la misma manera podemos extraer cualquier muestra de la seal x[n] si la multiplicamos por un
impulso desplazado, x[n]o[n-k]: Cuando n=k, o[n-k] es cero y, por consiguiente, x[n]o[n-k] tambin
es cero. Pero, para n=k, o[n-k] es uno y, por consiguiente, x[n]o[n-k] es x[k]. En consecuencia, la
seal {x[n]o[n-k], ne} es idntica a x[k] veces la seal {o[n-k], ne}:
{ } { } [ ] [ ], [ ] [ ], x n n k n x k n k n o o e = e
La siguiente figura muestra la combinacin lineal de todos estos impulsos desplazados,
{ } { } [ ], [ ] [ ],
k
x n n x k n k n o

=
e = e


-5 -4 -3 -2 -1 0 1 2 3 4
o[n]
o[n-1]
o[n-2]
o[n-3]
o[n-4]
u[n]
Universidad Distrital Francisco Jos de Caldas - Anlisis de Seales y Sistemas - Marco A. Alzate 52

Figura 47. Cualquier seal x[n] se puede representar como combinacin lineal de impulsos unitarios desplazados

Recordemos que estamos enfatizando la notacin orientada a la visin vectorial. En la mayora de
textos, estas expresiones se refieren a la manera de calcular la seal x[] en cada instante particular
n:
[ ] [ ] [ ],
k
x n x k n k n o

=
= e


Claro, una expansin semejante se puede hacer con base en el escaln unitario:
( ) [ ] [ ] [ 1] [ ]
k
x n x k x k u n k

=
=


pues, en efecto, para un n dado la suma slo considera valores de k menores o iguales a n, de
manera que la expansin resulta
[ ] [ ] [ 1] [ ] [ 1] [ 1] [ 2] [ 2] ... [ ]
n
k
x n x k x k x n x n x n x n x n x n
=
= = + + =


Si bien el impulso unitario en tiempo discreto cumple una funcin terica fundamental como base
ortogonal para la expansin del espacio vectorial de las seales en tiempo discreto, es importante
notar que tambin es una seal muy concreta que podemos generar y utilizar fcilmente en el
laboratorio. La contraparte en tiempo continuo, en cambio, tiene aplicaciones puramente tericas,
pues resulta imposible generarlo en el laboratorio. A continuacin definimos el escaln y el impulso
unitarios en tiempo continuo y mencionamos sus propiedades.

Partiendo del resultado que obtuvimos para el tiempo discreto,
-3 -2 -1 0 1 2 3 4
x[0]o[n]
x[1] o[n-1]
x[2] o[n-2]
x[3] o[n-3]
x[4] o[n-4]
x[n]
x[-1]o[n+1]
x[-2]o[n+2]
x[-3]o[n+3]
Universidad Distrital Francisco Jos de Caldas - Anlisis de Seales y Sistemas - Marco A. Alzate 53
0 0
[ ] [ ] [ ] [ 1]
1 0
n
u n n u n u n
n
o
<
= =

>


parece razonable definir sus contrapartes en tiempo continuo as:
0 0
( ) ( ) ( )
1 0
t
d
u t t u t
t dt
o
<
= =

>


aunque dicha definicin no est exenta de dificultades. Como el escaln unitario en tiempo
continuo, u(t), es constante en todas partes excepto en su punto de discontinuidad t=0, el impulso
unitario en tiempo continuo es cero en todas partes excepto en el punto de discontinuidad.
Formalmente, en ese punto el escaln no es derivable, as que tenemos un problema en la definicin
misma. Sin embargo, como de costumbre, lo que queremos hacer es una abstraccin matemtica
simplificada de alguna realidad compleja: Sabemos que la inercia de los sistemas reales nunca
permite respuestas con cambios que ocurren en instantes infinitesimales (perodos de tiempo de
longitud cero). Por ejemplo, el escaln unitario es una idealizacin del efecto de un interruptor
como el de la siguiente figura:

Figura 48. Respuestas ideal y real de un interruptor mecnico

Si el interruptor es ideal y la impedancia de entrada del circuito es infinita, el voltaje de entrada se
comportar exactamente como hemos definido el escaln unitario en tiempo continuo y como
aparece en la lnea roja punteada de la Figura 48. Pero si el interruptor rebota al accionarse y existen
impedancias considerables a la salida de la fuente y a la entrada del circuito, la situacin es ms
parecida a la lnea azul continua de la Figura 48. Tal vez podamos cambiar el interruptor mecnico
por un sofisticado mecanismo electrnico que produzca el siguiente voltaje a la entrada del circuito
lineal:

Figura 49. Forma ligeramente menos ideal del escaln unitario u
A
(t) y de su derivada, el impulso unitario o
A
(t)

En cuyo caso el escaln y el impulso unitarios ideales se podran definir como
0 0
( ) lim ( ) ( ) lim ( ) u t u t t t o o
A A
A A
= =
Circuito
lineal
+
-
+
u(t)
-
t=0
-0.5 0 0.5 1 1.5 2
0
0.5
1
tiempo en ms
u
(
t
)


real
ideal
u
(
t
)
Tiempo en s
u
A
(t)
t
t = 0 t = A
0
1
t
t = 0 t = A
0
1/A
( ) ( )
d
t u t
dt
o
A A
=
Universidad Distrital Francisco Jos de Caldas - Anlisis de Seales y Sistemas - Marco A. Alzate 54
El lmite de u
A
(t) es muy claro: La duracin del perodo de transicin, A, se va estrechando,
haciendo que, en el lmite, la transicin se vuelva instantnea. El lmite de o
A
(t), en cambio, necesita
un poco ms de reflexin. La siguiente figura muestra diferentes o
A
(t) para diferentes valores de A.

Figura 50. Algunas aproximaciones al impulso unitario en tiempo continuo

Dos propiedades evidentes de la aproximacin o
A
(t) son las siguientes:
| |
( ) 0, 0, ( ) 1 t t t dt o o

A A

= e A =
}

las cuales conducen, en el lmite cuando A0, a la definicin formal del impulso unitario en tiempo
continuo:
( )
( )
0 0
1
t
t
t dt
o
o

=

=

}

El intervalo de tiempo en el que el impulso unitario es diferente de cero, {te : o(t) =0}, es el
conjunto unitario {0}; Sin embargo, el rea debajo de la curva es uno. Claro, esto supone una
amplitud infinita de o(0), por lo que el impulso unitario en tiempo continuo se suele representar
como una flecha hacia el infinito, etiquetada con el rea debajo de ella, como muestra la siguiente
figura. Si el rea debajo de o(t) es uno, el rea debajo de ao(t) es a, tal como ocurre con ao
A
(t).

Figura 51. Representacin grfica del impulso unitario en tiempo continuo

A semejanza del impulso unitario en tiempo discreto, el impulso unitario en tiempo continuo resulta
muy til como abstraccin terica para desarrollar modelos muy interesantes de seales. Sin
0 1/8 1/4 1/2 1
0
8
4
2
1
t
o
A
(t)
o
1
(t)
o
1/2
(t)
o
1/4
(t)
o
1/8
(t)
t=0
o(t)
t
1
t=t
0
ao(t-t
0
)
t
a
t=0
Universidad Distrital Francisco Jos de Caldas - Anlisis de Seales y Sistemas - Marco A. Alzate 55
embargo, a diferencia del impulso unitario en tiempo discreto, el impulso unitario en tiempo
continuo no se puede construir en la realidad. Mientras el primero es una seal muy concreta que
podemos generar y manipular en el laboratorio (esto es, en el computador digital), el segundo es
solamente una idealizacin terica de la que no se puede disponer en el laboratorio. Pero,
considerando o(t) como el lmite de o
A
(t), podemos encontrar muchas de sus propiedades. Por
ejemplo, cmo es la seal x(t)o(t)? La siguiente figura, basada en aproximaciones o
A
(t), nos sugiere
la respuesta: Se trata de un impulso para el cual el rea debajo de la curva es x(0):

{x(t)o(t), te} = x(0){o(t), te}


Figura 52. Construccin para notar que x(t)o(t) es igual a x(0)o(t)

Claro, la figura anterior tambin nos permite imaginar el efecto de un desplazamiento en el tiempo:

{x(t)o(t-t
0
), te} = x(t
0
){o(t-t
0
), te}

que es la misma propiedad de seleccin que habamos encontrado para el impulso unitario en
tiempo discreto,
{ } { } [ ] [ ], [ ] [ ], x n n k n x k n k n o o e = e
Esta propiedad sugiere la posibilidad de combinar linealmente (en el tiempo continuo) una
secuencia de impulsos unitarios. En efecto, volviendo a la versin no idealizada, ntese que
{ } { } ( ), ( ) ( ),
k
x t t x k t k t o

A
=
e ~ A A e A


como muestra la siguiente figura.
0 1/8 1/4 1/2 1
t
x(t)o
A
(t)
o
1
(t)
o
1/2
(t)
o
1/4
(t)
o
1/8
(t)
x(t)1
x(t)2
x(t)4
x(t)8
0
8
4
2
1
Universidad Distrital Francisco Jos de Caldas - Anlisis de Seales y Sistemas - Marco A. Alzate 56

Figura 53. Construccin para notar que x(t) es una combinacin lineal (continua) de impulsos unitarios

A medida que A se vaya haciendo ms pequea, la aproximacin se va haciendo ms exacta hasta
que, en el lmite, obtenemos la igualdad:
{ } { } ( ), ( ) ( ), x t t x t t d t o t t

e = e
}

en analoga con la representacin que habamos encontrado para las seales en tiempo discreto,
{ } { } [ ], [ ] [ ],
k
x n n x k n k n o

=
e = e


Recordemos que estamos enfatizando la notacin orientada a la visin vectorial. En la mayora de
textos, estas expresiones se refieren a la manera de calcular las seales x() x[] en cada instante
particular de tiempo:
( ) ( ) ( ) , x t x t d t t o t t

= e
}

[ ] [ ] [ ],
k
x n x k n k n o

=
= e



La transicin infinitamente rpida del escaln unitario en tiempo continuo y la duracin
infinitesimal del impulso unitario en tiempo continuo son fenmenos que en matemticas se llaman
"singularidades": Puntos en los que hay discontinuidades, falta de diferenciabilidad o valores
infinitos, los cuales motivan toda un rea de las matemticas denominada "Teora de la
singularidad". Aunque parecen abstracciones tericas para nuestra experiencia cotidiana, en el
universo existen puntos de singularidad gravitacional (agujeros negros) o posibles instantes de
singularidad (como el big-bang). Para nuestro propsito "terrenal" del anlisis de seales, una
manera pragmtica de tratar las singularidades es considerando la aproximacin de intervalos de
longitud A que se van haciendo cada vez ms pequeos. Por ejemplo, considrese la siguiente seal
y su derivada:
t
( )
( ) ( )
k
x t
x k t k o

A
=
A A A

( ) ( ) x k t k o
A
A A A
( ) t k o
A
A
( ) t k o
A
A A
1/A
x(kA)
1
kA (k+1)A
Universidad Distrital Francisco Jos de Caldas - Anlisis de Seales y Sistemas - Marco A. Alzate 57

Figura 54. Seal con discontinuidades y su derivada
En los intervalos en los que x(t) permanece constante la derivada es cero. Igualmente, en los
intervalos en que x(t) crece o decrece a una tasa constante, la derivada toma el valor de la tasa de
crecimiento. Pero lo ms interesante es ver qu pasa en los puntos de discontinuidad. Como x(t) est
acumulando el rea debajo de la curva de su derivada, dicha derivada debe tener instantes con rea
apropiada para justificar el cambio instantneo del rea acumulada. Esto slo se puede explicar
mediante los impulsos o(t-1), -3o(t-3) y 2o(t-5), que se ubican en los puntos de discontinuidad de
x(t).

























t
0 1 2 3 4
0
1
2
-1
-2
-3
x(t)
6 5
t
0 1 2 3 4
0
1
2
-1
-2
-3
x(t)
6 5
1
2
-3
Universidad Distrital Francisco Jos de Caldas - Anlisis de Seales y Sistemas - Marco A. Alzate 58
Novena clase. Sistemas de procesamiento de seales

Como mencionamos en la tercera clase, las seales son cantidades fsicas medibles que existen en
un ambiente particular en el que se generan, se propagan, se almacenan y se transforman. Ese
ambiente que ejerce un proceso transformador en una seal se conoce como sistema. En efecto,
probablemente se trata de un conjunto de dispositivos o un conjunto de procesos que interactan
entre ellos para formar un todo (un sistema). En esa clase veamos, a manera de ejemplo, un circuito
formado por una resistencia y un condensador que forman un filtro pasabajos de primer orden, un
sistema mecnico formado por un resorte y una masa que forman un oscilador y un automvil que
se acelera bajo la accin de una fuerza. Todos ellos son sistemas en los que se puede identificar
seales de entrada, seales de salida, y procesos para generar las segundas a partir de las primeras.

La representacin matemtica del sistema, entonces, es la de un funcional que acepta como entrada
una de un conjunto de posibles seales de entrada y genera una de un conjunto de posibles seales
de salida:
T:XY
Donde X es el conjunto de las posibles seales de entrada y Y es el conjunto de posibles seales de
salida. Esto es, un sistema es un subconjunto del producto cartesiano XY. Es importante distinguir
entre la funcin (el modelo matemtico de la seal) y el funcional (el modelo matemtico del
sistema). Por ejemplo, en tiempo continuo, se podra representar la seal x(t) como una funcin que
a cada valor de la variable independiente te le asigna un valor de la variable dependiente xe.
De la misma manera, un sistema en tiempo continuo se puede representar como un funcional que
asigna una seal de salida {y(t), te} a cada seal de entrada {x(t), te}, como muestra la
siguiente figura:

Figura 55. Concepto de Seal como funcin y sistema como funcional

Desafortunadamente, la notacin grfica que se usa tpicamente en la literatura para los sistemas es
la que se muestra en la siguiente figura, la cual podra conducir a confusiones: No se trata de que,
para cada instante de tiempo t, el valor especfico y(t) dependa del valor especfico x(t); se trata de
que la seal entera {y(t), te} depende de la seal entera {x(t), te}, de manera que el valor
especfico y(t) puede depender de muchos valores {x(t), teT_}. Como veremos, las posibilidades
incluyen que, para un valor de t
0
dado, el valor especfico y(t
0
) slo dependa de x(t
0
), en cuyo caso
se dice que el sistema no tiene memoria y la figura 56 cobrara sentido; o puede ser que y(t
0
)
dependa de todos los valores de x en instantes no superiores a t
0
, {x(t), tst
0
}, en cuyo caso se dice
que el sistema es causal. Sin embargo, es explicable que se prefiera la representacin de la figura 56
{x(t)e, te}eX {y(t)e, te}eY
te xe
Seal : Funcin que
asigna a cada valor de
entrada te un
correspondiente valor de
salida x(t)
Sistema : Funcional que
asigna a cada seal de
entrada {x(t)e, te}
una correspondiente seal
de salida {y(t)e, te}
Universidad Distrital Francisco Jos de Caldas - Anlisis de Seales y Sistemas - Marco A. Alzate 59
porque la de la figura 55 puede resultar engorrosa cuando el sistema se compone de muchos
subsistemas. Como en la mayora de la literatura, en el resto de este curso el contexto dir si la
expresin x(t) se refiere al valor particular de la seal {x(t), te} en el instante t, o si se refiere de
manera genrica a toda la seal {x(t), te}. Cuando, por alguna razn, en el segundo caso
necesitemos ser especficos, nos referiremos a la seal completa como {x(t), te} o como {x(t)}
t
, y
al valor particular como x(t). Y para referirnos a la transformacin que hace un sistema en la seal
de entrada {x(t)}
t
eX, para producir una seal de salida {y(t)}
t
eY, hablaremos de la
transformacin {y(t)}
t
= T({x(t)}
t
) o, a veces, hablaremos del par entrada/salida ({x(t)}
t
, {y(t)}
t
).


Figura 56. Notacin tpica para un sistema en tiempo continuo. Tiende a hacer perder de
vista que se trata de un funcional y no de una funcin

Excepto por el tipo de sistemas que veremos en la vigsimo sexta clase, en este curso
consideraremos slo dos tipos de sistemas: Sistemas en tiempo continuo (aceptan, procesan y
generan seales en tiempo continuo) y sistemas en tiempo discreto (aceptan, procesan y generan
seales en tiempo discreto), como se muestra en la siguiente figura.

Figura 57. Los dos tipos de sistemas que se consideran en este curso (a excepcin de la vigsimo sexta clase)

Ntese que hemos descrito los sistemas mediante la relacin que existe entre la seal de entrada y la
seal de salida, con lo cual ignoramos los detalles internos del sistema. En otros contextos puede ser
posible conocer algunos aspectos de la construccin interna del sistema y reconocer que existen
otras seales que describen el estado del sistema en cada instante, de manera que la descripcin del
sistema incluye la relacin que existe entre la seal de entrada y el estado interno del sistema
(ecuaciones de estado) y la relacin que existe entre la seal de salida y las seales de estado interno
y de entrada (ecuaciones de salida). La diferencia entre la descripcin de los sistemas en el espacio
de estados y la descripcin como relacin entrada/salida es una diferencia conceptual: En el primer
caso conocemos la estructura interna del sistema (modelo de "caja blanca") y en el segundo caso la
desconocemos o decidimos ignorarla (modelo de "caja negra"). En este curso enfatizaremos el
modelo de caja negra, aunque en el momento en que hagamos diseo de sistemas de procesamiento
de seales deberemos conocer perfectamente bien su estructura interna (es nuestro diseo!) y
usaremos modelos de caja blanca (aunque tal vez no seremos explcitos en mencionarlo).

x(t) y(t)
{x(t)e, te}eX {y(t)e, te}eY
{x[n]e, ne}eX {y[n]e, ne}eY
Sistema en
tiempo
continuo
Sistema en
tiempo
discreto
Universidad Distrital Francisco Jos de Caldas - Anlisis de Seales y Sistemas - Marco A. Alzate 60
Ahora veremos 6 propiedades particulares de los sistemas en general que nos permitirn
clasificarlos y escoger algunos tipos de sistemas de mayor inters para su anlisis detallado:

(1) sistemas con y sin memoria (o estticos y dinmicos),
(2) sistemas invertibles y no invertibles,
(3) sistemas causales y no causales,
(4) sistemas bibo-estables o no bibo-estables,
(5) sistemas invariantes o no invariantes en el tiempo y
(6) sistemas lineales o no lineales.

Como las seis propiedades resultan completamente anlogas entre sistemas en tiempo discreto y
sistemas en tiempo continuo, usaremos indistintamente uno u otro tipo de seales para la definicin
y para los ejemplos. Si usamos el tiempo continuo, el estudiante podr redefinir los conceptos en el
tiempo discreto y viceversa.

(1) Un sistema carece de memoria (o es un sistema esttico) si el valor de la seal de salida en cada
instante slo depende del valor de la seal de entrada en ese mismo instante. Por ejemplo, en una
resistencia ideal de R ohmios en la que la seal de entrada es el voltaje que se aplica a sus
terminales, x(t), y la seal de salida es la corriente que circula a travs de la resistencia, y(t),
tenemos la siguiente relacin entre la seal de salida y la seal de entrada:

Figura 58. Ejemplo de un sistema sin memoria

Como el valor instantneo de la corriente slo depende del valor del voltaje en ese mismo instante y
no depende de valores pasados (ni futuros) del voltaje, el sistema no tiene memoria (es esttico).
Los sistemas sin memoria se pueden modelar como una funcin y=f(x), no necesariamente como un
funcional {y(t), te}=T({x(t), te}). Claro, como la entrada es una seal, la salida una funcin del
tiempo, y(t) = f(x(t)), por lo que se trata de un funcional degenerado.

En un sistema con memoria, o dinmico, la seal de salida en un instante particular de tiempo
depende de los valores de la entrada (o de salida) en diferentes instantes de tiempo. Es decir, el
sistema debe "recordar" algunos valores anteriores (o futuros!) de la seal de entrada y/o de la seal
de salida. Por ejemplo, en una bobina ideal de L henrios en la que la seal de entrada es el voltaje
que se aplica a sus terminales, x(t), y la seal de salida es la corriente que circula a travs de la
resistencia, y(t), tenemos la siguiente relacin entre la seal de salida y la seal de entrada:

Figura 59. Ejemplo de un sistema con memoria
R
x(t)
y(t)
+
-
y(t)=x(t)/R
L
x(t)
y(t)
+
-
0
1
( ) ( ) y t x t d
L
t t

=
}
Universidad Distrital Francisco Jos de Caldas - Anlisis de Seales y Sistemas - Marco A. Alzate 61
Como el valor instantneo de la corriente no slo depende del valor del voltaje en ese mismo
instante sino de todos los valores anteriores del voltaje, se dice que el sistema tiene memoria. En un
sistema con memoria se requiere de un dispositivo que almacene informacin anterior. Por ejemplo,
un condensador recuerda el voltaje infinitesimalmente anterior, una bobina recuerda la corriente
infinitesimalmente anterior y un flip-flop tipo D recuerda el bit de entrada que haba en el pulso de
reloj inmediatamente anterior. En efecto, una versin de tiempo discreto de la bobina anterior sera
el de un sistema acumulador:
0
[ ] [ ]
k
y n x n k

=
=


que parece necesitar una memoria infinita pues para calcular y[n] se necesitan todas las muestras
anteriores de la seal de entrada desde x[-] hasta x[n]. Pero en realidad la bobina almacena toda
la seal de voltaje de entrada en su memoria? No. No hace falta. Por ejemplo, el acumulador slo
debe almacenar el ltimo acumulado:
0 1 0
[ ] [ ] [ ] [ ] [ ] [( 1) ] [ ] [ 1]
k k k
y n x n k x n x n k x n x n k x n y n

= = =
= = + = + = +



Figura 60. Acumulador: Otro ejemplo de un sistema con memoria

De la misma manera que el acumulador slo debe recordar la salida inmediatamente anterior, x[n] =
y[n] y[n-1], la bobina slo debe recordar la corriente inmediatamente anterior
( ) ( )
d
x t L y t
dt
=
A veces un sistema con memoria debe "recordar" el futuro. Por ejemplo, en procesamiento de
imgenes existen procesos en los que el nuevo valor de un pixel particular se determina al
compararlo con los valores de los pixeles vecinos, donde la seleccin del vecindario (elemento
estructurante) determina el efecto final de la operacin. Por ejemplo, la operacin

y(i,j) = mx{x(i-1,j), x(i,j), x(i+1,j), x(i,j-1), x(i,j+1)} - mn{x(i-1,j), x(i,j), x(i+1,j), x(i,j-1), x(i,j+1)}

no slo usa el presente (x(i,j)) sino que tiene memoria del "pasado" (x(i-1,j), x(i,j-1)) y tambin del
"futuro" (x(i+1,j), x(i,j+1)). Los efectos de dicha operacin son muy interesantes:
x[n] E y[n]
Retardo
y[n-1]
+
+
Universidad Distrital Francisco Jos de Caldas - Anlisis de Seales y Sistemas - Marco A. Alzate 62

Figura 61. Resultado de un proceso con memoria del pasado y del futuro
La presencia o ausencia de memoria se puede determinar por inspeccin: Ver si en la expresin que
relaciona la seal de salida en un instante t con la seal de entrada o salida en instantes anteriores.
Por ejemplo, y[n] = x[n] + (n-1) es un sistema sin memoria a pesar del trmino (n-1), pues dicho
trmino no implica la necesidad de conocer ningn dato anterior. Otra cosa ocurre con el sistema
y[n] = x[n-1] + (n), pues ahora el trmino (n-1) s est indicando la necesidad de recordar el valor de
la seal de entrada en el instante inmediatamente anterior.

(2) Un sistema es invertible si para cada seal de salida se puede identificar unvocamente la seal
de entrada que la gener. Siendo as, podramos construir (al menos en principio) un sistema
inverso que recupere la seal de entrada a partir de la seal de salida: Si {y(t)}
t
= T({x(t)}
t
) es un
sistema invertible, existe un sistema T
-1
tal que {x(t)}
t
= T
-1
({y(t)}
t
). Por ejemplo, como acabamos
de ver, el acumulador es invertible y su sistema inverso, el diferenciador, es muy fcil de construir:

Figura 62. El diferenciador es el sistema inverso del acumulador

Efectivamente, si ponemos el acumulador de la figura 61 en serie con el diferenciador de la figura
62, el resultado final ser el sistema identidad: La entrada es idntica a la salida. En cambio, el
sistema de la siguiente figura no es invertible pues diferentes seales de entrada pueden producir la
misma seal de salida.

Figura 63. Sistema no invertible: Diferentes seales de entrada producen la misma seal de salida

Determinar la invertibilidad de un sistema consiste en encontrar el sistema inverso (si el sistema es
invertible) o en demostrar que no existe un sistema inverso (por ejemplo, encontrando dos seales
de entrada que produzcan la misma salida). Por ejemplo, consideremos los siguientes dos sistemas:
{y(t)}
te = {x(2t)}
te y {y[n]}
ne = {x[2n]}
ne. En el primero, el sistema inverso es fcil de
encontrar: x(t) = y(t/2), lo cual nos da exactamente la seal de entrada que origin y(t). En el
Seal de entrada
x:{1,2,,500} {1,2,,600}{0,1}
Seal de salida
y(i,j) = mx{x(i-1,j), x(i,j), x(i+1,j), x(i,j-1), x(i,j+1)}
- mn{x(i-1,j), x(i,j), x(i+1,j), x(i,j-1), x(i,j+1)}
y[n] E x[n]
Retardo
x[n-1]
+
-
+
x(t)

+
y(t)

Universidad Distrital Francisco Jos de Caldas - Anlisis de Seales y Sistemas - Marco A. Alzate 63
segundo sistema, alguien podra sugerir que el sistema inverso se puede definir similarmente, x[n] =
y[n/2], pero en este caso queda la ambigedad de qu poner en las muestras impares de x[], pues la
relacin es vlida slo para valores pares de n. De hecho, las siguientes dos seales de entrada
producen la misma seal de salida:

Figura 64. El sub-muestreador no es un sistema invertible

(3) Como vimos antes, un sistema con memoria puede "recordar" el futuro, esto es, puede que la
seal de salida en el instante t dependa de valores de las seales de entrada o salida posteriores al
instante t. En ese caso, se trata de un sistema no causal. Pero si, para calcular el valor de la seal de
salida en cualquier instante, slo hace falta conocer el valor actual y, posiblemente, valores pasados
de las seales de entrada o de salida, entonces se trata de un sistema causal.

Ms formalmente, sea ({x
1
(t), te}, {y
1
(t), te}) e XY cualquier par de seales entrada/salida
de un sistema dado, y sea ({x
2
(t), te}, {y
2
(t), te}) e XY otro par diferente de seales
entrada/salida del mismo sistema, con la propiedad de que existe un t
0
para el cual x
1
(t) = x
2
(t)
tst
0
. El sistema es causal cuando esta condicin implica que y
1
(t) = y
2
(t) tst
0
.

Los sistemas en que la variable independiente es realmente el tiempo y que deben trabajar en
tiempo real DEBEN ser causales. La no causalidad es para sistemas en los que la variable
independiente no es el tiempo (como el sistema de procesamiento de imgenes de la figura 61) o
para los que el procesamiento no se hace en tiempo real. Considere, por ejemplo, el procesamiento
que se hace para mejorar la calidad de audio durante la transferencia de msica desde cintas de
audio analgicas hacia formatos digitales: en estas condiciones no habra razn para limitarnos a
usar sistemas causales. Por ejemplo el siguiente sistema causal calcula el promedio de las 11
ltimas muestras de la entrada:
10
0
1
[ ] [ ]
11
k
y n x n k
=
=


Pero, si pudisemos usar un sistema no causal, tal vez sera preferible usar el siguiente sistema:
5
5
1
'[ ] [ ]
11
k
y n x n k
=
=


Ntese que, en este caso, la nica diferencia entre las seales de salida y[n] y y'[n] es un retardo de 5
unidades de tiempo.

Para determinar la causalidad debemos determinar si la variable de salida en cualquier instante es
independiente de valores futuros de la seal de entrada (o de salida). Por ejemplo, la seal
y(t) = x(-t) podra parecer causal porque, para cualquier t > 0, la salida slo depende de muestras
0 1 2 3 4 5 6 7 8 9
0 1 2 3 4 5 6 7 8 9
n
x
1
[n]
n
x
2
[n]
0 1 2 3 4
n
y
1
[n]
0 1 2 3 4
n
y
2
[n]
y[n]=x[2n]
y[n]=x[2n]
Universidad Distrital Francisco Jos de Caldas - Anlisis de Seales y Sistemas - Marco A. Alzate 64
anteriores de la entrada. Pero, por la misma razn, para t < 0 la salida depende de valores futuros de
la entrada. En consecuencia, y(t) = x(-t) es una seal no causal.

(4) La estabilidad es un concepto relativamente elaborado en el estudio de sistemas dinmicos. En
particular, si usamos un modelo de espacio de estados para describir el sistema, podemos definir
muchas formas de estabilidad dependiendo de cmo responde el sistema a estados internos iniciales
cuando la entrada es cero (estabilidad uniforme, estabilidad exponencial, estabilidad asinttica,
etc.). En este curso usamos modelo de entrada/salida para describir los sistemas, lo cuales no
consideran explcitamente posibles estados internos del sistema, por lo que duchos conceptos no
aplican directamente. Por eso el caso de estabilidad que consideraremos es el ms general posible:
Estabilidad BIBO (Bounded-Input, Bounded-Otuput): Si la seal de entrada es acotada, la seal de
salida tambin es acotada:
{ } par entrada/salida ( ( ), ( )), tal que ,
x y
x t y t t x M y M

e s < s <
(El trmino (x(

quiere decir max ( )


t
x t
e
, como se describe en la siguiente clase).
El hecho de que no consideremos estados internos del sistema implica que la condicin BIBO ser
suficiente criterio de estabilidad para nosotros aunque por dentro algo huela a quemado!

Determinar la estabilidad BIBO de un sistema consiste en encontrar una cota en el valor mximo de
{y(t)}
t
dada la cota en el valor mximo de {x(t)}
t
, o demostrar que dicha cota no existe. Por
ejemplo, el acumulador y[n] = x[n] + y[n-1] no es estable porque basta con notar que para x[n]=u[n],
acotada con (x(

s 1, la salida es y[n] = n+1, que crece sin lmite. Pero si el sistema incluye una
ponderacin de las muestras, y[n] = ox[n] + (1-o)y[n-1], con 0<o<1, entonces
0
[ ] (1 ) [ ]
k
k
y n x n k o o

=
=

de manera que, si
0 0
, [ ] (1 ) [ ] (1 )
k k
k k
x M y n x n k M M o o o o

= =
s s s =

.

(5) Un sistema es invariante en el tiempo si para cualquier par de seales de entrada/salida (x(t),
y(t)) y para cualquier desplazamiento de tiempo t
0
, las seales (x(t-t
0
), y(t-t
0
)) tambin forman un par
entrada/salida del sistema. Esto es, un desplazamiento en el tiempo de la seal de entrada origina el
mismo desplazamiento en el tiempo de la seal de salida. Fundamentalmente, esta propiedad se
refiere a que los parmetros del sistema no cambian con el tiempo. Si llegamos al laboratorio a las
3:00 pm a hacer un experimento con el regulador zener de la figura 63, obtendremos los mismos
resultados que obtendramos si tenemos un retraso y llegamos a las 3:20 pm. Esto se debe a que ni
las caractersticas de la resistencia ni las caractersticas del diodo cambiarn en ese lapso de 20
minutos.

Para determinar si un sistema es invariante en el tiempo, partimos de un par genrico de seales
entrada/salida ({x(t), te}, {y(t), te}) y miramos el respectivo par para la misma entrada
desplazada ({x
to
(t) = x(t-t
0
), te}, {y
to
(t), te}). Si, al comparar las dos salidas, notamos que y
to
(t)
= y(t-t
0
) para todo te, sabremos que el sistema es invariante. De otra manera, sabremos que el
sistema no es invariante. Por ejemplo, en el circuito RC de la figura 8, supongamos que ({x(t),
te}, {y(t), te}) es un par de seales que satisfacen la relacin x(t) = y(t) + RC dy(t)/dt. Si ahora
introducimos x
to
(t) = x(t-t
0
), la salida y
to
(t) debe satisfacer x(t-t
0
) = y
to
(t) + RC dy
to
(t)/dt. Si
remplazamos y(t-t
0
) en vez de y
to
(t) y hacemos el cambio de variable s = t-t
0
, obtenemos la ecuacin
Universidad Distrital Francisco Jos de Caldas - Anlisis de Seales y Sistemas - Marco A. Alzate 65
x(s) = y(s) + RC dy(s)/ds, ya que dy(t-t
0
)/dt = dy(s)/ds en s=t-t
0
. Y, como ya sabamos que y(s)
satisface esta ecuacin, se demuestra que el sistema es invariante. Sin embargo, si ahora la
resistencia cambia con el tiempo de manera que en el instante t vale R(t) ohmios, la relacin original
x(t-t
0
) = y
to
(t) + R(t)C dy
to
(t)/dt al remplazar y(t-t
0
) en vez de y
to
(t) y s en vez de t-t
0
queda
x(s) = y(s) + R(s+t
0
)C dy(s)/ds, que no es la relacin satisfecha originalmente por y(s). El hecho de
que los parmetros del sistema cambien con el tiempo hace que el sistema no sea invariante.

Como ejemplo adicional, considere nuevamente el sistema y(t) = x(2t). Si introducimos la seal
x
to
(t)=x(t-t
0
) obtenemos y
to
(t) = x
to
(2t) = x(2t t
0
). Pero y(t-t
0
) = x(2(t-t
0
)) = y
to
(t), por lo que el
sistema no es invariante en el tiempo. La siguiente figura ilustra este efecto con un ejemplo:


Figura 65. La invarianza en el tiempo no es una propiedad de la escalizacin en el tiempo

(6) Un sistema lineal satisface la propiedad de superposicin segn la cual, para cualquier par de
entradas/salidas {y
1
(t)}
t
= T({x
1
(t)}
t
) y {y
2
(t)}
t
= T({x
2
(t)}
t
) y cualquier par de escalares o y |, se
satisface que {oy
1
(t) + |y
2
(t)}
t
= T({ox
1
(t) + |x
2
(t)}
t
). A veces esta propiedad se divide en dos:
aditividad (T({x
1
(t) + x
2
(t)}
t
) = T({x
1
(t)}
t
) + T({x
2
(t)}
t
)), y homogeneidad (T({ox
1
(t)}
t
) =
oT({x
1
(t)}
t
)). Aplicando estas propiedades inductivamente, si {x
i
(t)}
te es una secuencia de seales
de entrada y si a
i
es una secuencia de escalares, el principio de superposicin se extiende as:
{ } ( )
1 1
( ) ( )
i i i i
t
i i
t
T a x t aT x t

e
= =
e
| |

=
` |
)
\ .


Esta propiedad tiene importantes consecuencias para los sistemas lineales, las cuales los hacen muy
fciles de analizar y disear. Por ejemplo, ntese que si la entrada es x(t)0 te, la salida tiene
que ser igualmente y(t)0 te. En efecto, para cualquier otra seal diferente de cero, por ejemplo
o(t), se tiene que x(t) = 0o(t) y, por consiguiente, {y(t)}
t
= T(0{o(t)}
t
))= 0T({o(t)}
t
) 0 te.

Para determinar si un sistema es lineal o no, se aplica directamente la definicin verificando si se
satisface el principio de superposicin. Por ejemplo, el sistema sin memoria y(t) = x(t) + 1 es,
obviamente, no lineal porque cuando la entrada es idnticamente cero la salida no es idnticamente
cero. Pero podemos hacer la verificacin desde la definicin: si introducimos ax
1
(t) + bx
2
(t) a la
entrada, obtenemos a la salida y(t) = ax
1
(t) + bx
2
(t) + 1. Sin embargo, la combinacin lineal de las
salidas individuales es a(x
1
(t) + 1) + b(x
2
(t) + 1) = y(t) + a + b 1.

Una clase muy importante de sistemas son los lineales e invariantes en el tiempo. Aunque no son
muy comunes en la naturaleza, se facilita tanto su anlisis y es tan extensa su teora que resulta de
0 1 2
0
1
0 1 2
0
1
3
3
x(t)
t
x(t)=x(t-1)
t
y(t)=x(2t)
y(t)=x(2t)
0 1 2
0
1
3
y(t), y(t-1)
t
0 1 2
0
1
3
y(t), y(t-1)
t
Universidad Distrital Francisco Jos de Caldas - Anlisis de Seales y Sistemas - Marco A. Alzate 66
suma importancia estudiarlos con gran cuidado. Por un lado, si el sistema ya est dado, podran
llegar a encontrarse aproximaciones lineales, vlidas al menos en contextos restringidos de
operacin del sistema. Pero si el sistema debe ser diseado y construido por nosotros mismos, las
enormes ventajas que traen la linealidad y la invarianza hacen que valga la pena intentar dotar al
sistema de estas caractersticas cuando sea posible. En efecto, existen poderosas herramientas
tericas para el anlisis y la sntesis de este tipo de sistemas, ya que las dos propiedades hacen que
estos sistemas se caractericen por una nica seal: su respuesta al impulso. De alguna manera, el
anlisis y la sntesis de sistemas lineales e invariantes se reducen al anlisis y el diseo de la seal
de respuesta al impulso.

Sin embargo, antes de empezar el estudio formal de los sistemas lineales e invariantes, y dada la
informacin introductoria que ya hemos presentado, es oportuno ofrecer un panorama del contexto
terico en que se apoya el anlisis de seales y sistemas lineales e invariantes: La teora de espacios
vectoriales abstractos.






























Universidad Distrital Francisco Jos de Caldas - Anlisis de Seales y Sistemas - Marco A. Alzate 67
Tercera Tarea

1. Sea x(t) la exponencial compleja en tiempo continuo, e
j2tt/T
. Considere la seal en tiempo discreto que se obtiene al
tomar valores de x(t) cada At segundos, esto es, x[n] = x(nAt). Encuentre la condicin ms general que debe
satisfacer At para que x[n] sea una seal peridica.

2. Sean x
1
[n] y x
2
[n] seales peridicas en tiempo discreto con perodos fundamentales N
1
y N
2
respectivamente.
Cules son las condiciones ms generales para que la suma x[n] = x
1
[n] + x
2
[n] sea una seal peridica? Cul es
el perodo fundamental de x[n] bajo esas condiciones?

3. Sea la seal en tiempo continuo x(t)=cos(et), con e = t/4. Considere 12 seales en tiempo discreto obtenidas al
tomar muestras de x(t) cada At segundos con At e {1, 2, , 12}. Dibuje 16 muestras de cada seal discreta (use
stem). Superponga la respectiva seal en tiempo continuo (use plot con un rango muy denso de t en el intervalo
correspondiente a las 16 muestras). En todos los casos, use en el eje horizontal el nmero de muestra de 0 a 15. (En
una pgina puede presentar todos los resultados si los ordena en una matriz de 43 grficas, por ejemplo). Cul es
la frecuencia, en ciclos por muestra, de cada seal discreta? Cul es el perodo? Concluya discutiendo los aspectos
que este ejercicio le recuerda sobre la periodicidad en tiempo discreto.

4. Determine si las siguientes seales son peridicas o aperidicas y, en el primer caso, determine su perodo
fundamental. Encuentre las componentes par e impar de cada seal. Clasifique cada seal segn su potencia y su
energa.
5
2
1
5
2
( ) ( ) cos (2 ) ( ) ( ) ( 2 )
( ) ( ) 2 (3 ) ( ) [ ] ( 1)
( ) [ ] cos( / 4) ( ) [ ] cos( / 4)
( ) ( ) 5cos( / 4) sin(5 / 3) ( ) [ ] 5cos( / 4) sin(5 / 3)
k
n
k
a x t t b x t T t k
c x t T k t d x n
e x n n f x n n
g x t t t h x n n n
t
t
t t t t
=

=
= =
= =
= =
= + = +


donde T
1
(t) es un pulso triangular entre -1 y 1 con altura 1, y T
2
(t) es un pulso triangular entre 0 y 2, con altura 1.

5. (a) Demuestre que o(ot)=o(t)/|o| (b) Demuestre que si x(t) tiene ceros sencillos en {t
1
, t
2
, , t
n
}, y en esos puntos
la derivada x'(t
i
) existe y no es cero, entonces o(x(t))=E
i=1..n
o(t-t
i
)/|x'(t
i
)|

6. Determine si los siguientes sistemas son (a) con memoria o sin memoria, (b) causales o no causales, (c) invertibles
o no invertibles, (d) BIBO estables o no BIBO estables, (e) lineales o no lineales (f) invariantes en el tiempo o no
invariantes en el tiempo.
a. y[n] = x[2-n]
b. y(t)=x(t)/[1-x(t-1)]
c. y(t)=tx(t)
d. y[n]=E
k=-0
x[n-k]
e. y(t)=[d/dt]x(t)
f. y(t)=t/[1+x(t-1)]
g. y[n]=x[n]+nx[n-1]
h. y(t)=x(sen(t))
i. y[n]=x[n]x[n-2]
j. y[n]=E
k=-mm
x[n-k]
k. y[n]=x[n-1]+x[n+1]
l. El sistema compuesto por la conexin en serie de los dos ltimo sistemas (j y k)
m. y(t)=x(t-2)+x(2-t)
n. y(t)=x(t/3)
o. y[n]=x[4n+1]
p. y[n]=Componente par de x[n-1]
Universidad Distrital Francisco Jos de Caldas - Anlisis de Seales y Sistemas - Marco A. Alzate 68
Dcima clase. Repaso de lgebra lineal: Visin geomtrica de las seales y los sistemas.

Espacios Vectoriales Euclidianos. Independencia lineal. Sub-espacios. Matrices. Rango.
Determinantes. Menores. Transpuesta. Inversa. Ecuaciones lineales. Producto interno. Normas.
Desigualdad de Cauchy-Schwarz. Desigualdad del tringulo. Transformaciones lineales.
Eigenvalores y eigenvectores. Proyecciones ortogonales. Espacios Vectoriales Abstractos. Espacios
Normados. Espacios de Producto Interno. Espacios de Hilbert.

A la luz de estos conceptos, notar que las seales de energa, las seales de potencia, las seales
pares, las seales impares, y las seales peridicas forman sub-espacios vectoriales. Definir la
correlacin en tiempo continuo y describir la energa como el producto interno de un vector consigo
mismo. Explicar por qu la correlacin se usa para comparar seales. Notar la conveniencia de
expandir seales en componentes perpendiculares. Notar que las muestras de una seal son su
expansin en la base dada por los desplazamientos del impulso unitario, que es una base
ortonormal. Extender al espacio de las seales en tiempo continuo. Notar que la expansin de
seales peridicas como combinacin lineal de exponenciales complejas armnicamente
relacionadas es simplemente un cambio de base del mismo espacio vectorial donde los coeficientes
de la expansin son los nuevos componentes de cada vector.


























Universidad Distrital Francisco Jos de Caldas - Anlisis de Seales y Sistemas - Marco A. Alzate 69
Solucin de la primera tarea
/ /
1
Durante los primeros 30 ms, = . Sea ( ) 1 ( ) de manera que , donde se pueden
1
separar las variables para obtener , de donde ( ) ( ) 1 . Como
(0) 0, entonc
t t
dy y du u
u t y t
dt dt
du
dt u t Ce y t Ce
u
y
o o
o o
o

= =
= = =
=
} }
/
-( -0.03)/ 3
es 1. En consecuencia, ( ) 1 , 0 0.03. De la misma manera se puede
encontrar que, a partir de 0.03, ( ) (0.03) , donde (0.03) 1 0.9502. Entonces

t
t
C y t e t
t y t y e y e
o
o

= = s s
= = = ~
100
(100 3)
0 0
( ) 1 0 0.03
0.9502 0.03
t
t
si t
y t e si t
e si t


<

= s <

>


( ) ( ) ( )
2
2 2 3
0
1 1
De otro lado, [ ] [ ] [ 1] se puede expandir as: [0] [0],
1 1 1
1 1
[1] [1] [0], [2] [2] [1] [0], y, en
1 1
1 1 1
1 1
general, [ ] [ ]
1 1 1
d d d d d
d d d d d d d
k
n
d d
k
y n x n y n y x
y x x y x x x
y n x n k
|
| | |
| | |
| |
| | |
|
| |
=
= + =
+ + +
= + = + +
+ +
+ + +
| |
= =
|
+ + +
\ .

( )
( )
0
1
1
max(0, 30)
max 0, 30
/(1 )
1
30 1
0 30
1
0 0
1 1 1
[ ]
0 1 1 1 1 1
0 0
[ ] 1 0 30
k
n
k
k
a n a n n
k
d a n
a n
k n k a
n
d
n n
n k
n
y n
n
si n
y n si n
s
| |
|
| |
|

| | | |


=
+
+
=
= =
= +
+
+
| |
s s
|
+
\ .
< | |
= = = =

|
> + + + +
\ .
<
= s s


1
10/11
30 i n
=

>


Al muestrear 5 veces ms rpido, tendramos
1
150 1
50/51
0 0
'[ ] 1 0 150
150
n
d
n n
si n
y n si n
si n
t
t
t t
+
+
=
<

= s s

>


Los respectivos MSE son:
( ) ( ) ( )
( ) ( ) ( )
80 30 80
2 2
2
1 /10 /10
0 0 31
400 150 400
2 2
2
1 /50 /50
0 0 151
1 1
(1 ms) ( / 1000) [ ] 19.085537 16.540311
81 81
1 1
(0.2 ms) ( / 5000) '[ ] 19.085537 18.519211
401 401
n n n n
d
n n n
n n n n
d
n n n
MSE y n y n e e
MSE y n y n e e

t t
+
= = =
+
= = =
(
= = +
(

= = +


(
(


Se puede evaluar fcilmente las expresiones anteriores para y(t), y
d
[n] y y
d
[n] utilizando desde una
calculadora de bolsillo hasta un computador personal que cuente con una aplicacin de hoja
electrnica como Excel o con un lenguaje de programacin general como Java o C++. En este
curso usaremos Matlab, pues para aplicaciones de ingeniera es til aprender a usar software de
computacin cientfica y Matlab es uno de los ms poderosos y utilizados (ver quinta clase). El
siguiente script en Matlab evala dichas expresiones.

% Calcula la salida del sistema en tiempo discreto con dt = 1 ms
n=-10:80; % Indice de tiempo discreto (intervalos de 1 ms)
r = 10/11; % Razn beta/(1+beta)
yd = (n>=0).*(n<=30).*(1-r.^(n+1)) + (n>30).*(r.^(n-30) - r.^(n+1));
% Obtiene las correspondientes muestras de la salida del sistema en tiempo
% continuo y calcula el error cuadrado medio
t = n/1000;
yt = (t>=0).*(t<=0.03).*(1-exp(-100*t)) + (1-exp(-3))*(t>0.03).*exp(-(100*t-3));
Universidad Distrital Francisco Jos de Caldas - Anlisis de Seales y Sistemas - Marco A. Alzate 70
MSE1 = sum((yd(11:91) yt(11:91)).^2)/81
% Calcula la salida del sistema en tiempo discreto con dt = 0.2 ms
m=-50:400; % Nuevo ndice de tiempo discreto (intervalos de 0.2 ms)
r = 50/51; % Nueva razn beta/(1+beta)
yp = (m>=0).*(m<=150).*(1-r.^(m+1)) + (m>150).*(r.^(m-150) - r.^(m+1));
% Obtiene las correspondientes muestras de la salida del sistema en tiempo
% continuo y calcula el error cuadrado medio
t = m/5000;
yt = (t>=0).*(t<=0.03).*(1-exp(-100*t)) + (1-exp(-3))*(t>0.03).*exp(-(100*t-3));
MSE2 = sum((yp(51:451) yt(51:451)).^2)/401
% Grafica las tres seales
stem(m/5000,yp,'k.')
hold on
stem(n/1000,yd,'ro')
hold on
plot(t,yt,'r-')
legend('y_d a 0.2 ms','y_d a 1 ms','y(t)')

Las seales obtenidas se muestran a continuacin, junto con las medidas del error cuadrado
promedio:

MSE
1 ms
= 437.2510
-6
MSE
0.2 ms
= 17.9910
-6


Al disminuir el perodo de muestreo cinco veces, el error cuadrado medio disminuy ms de 24
veces pues, en efecto, la aproximacin de la derivada como el cociente de incrementos en la seal
con respecto al perodo de muestreo se hace menos imprecisa. Pero lo ms interesante de este
ejercicio es haber observado el tratamiento analtico de seales en tiempo continuo y en tiempo
discreto mediante abstracciones matemticas que pueden representar fenmenos reales y su
evaluacin computacional. En particular, evaluando directamente los resultados analticos pudimos
aprovechar el manejo de vectores de matlab, pero tambin es posible evaluar directamente el
proceso descrito en tiempo discreto, aunque nos toque usar el odioso lazo for:

n = -10:80;
x = (n>=0).*(n<=30);
y = zeros(size(x));
for m=-10:80
k = m+11;
if m<0
y(k) = 0;
else
y(k) = (x(k)+10*y(k-1))/11;
end
end
hold on
stem(n/1000,y,'gx')


-0.01 0 0.01 0.02 0.03 0.04 0.05 0.06 0.07 0.08
0
0.1
0.2
0.3
0.4
0.5
0.6
0.7
0.8
0.9
1


y
d
a 0.2 ms
y
d
a 1 ms
y(t)
Universidad Distrital Francisco Jos de Caldas - Anlisis de Seales y Sistemas - Marco A. Alzate 71
Solucin de la segunda tarea

1. Sea N el perodo de la seal {x[n], ne}. Por definicin,
2 1
lim [ ]
2 1
M
M
n M
Px x n
M

=
=
+

. Como el
lmite es el mismo independientemente de la manera como acerquemos M a , podemos usar
valores de M que involucren nmeros enteros de perodos:
( ) ( )
| | | |
1 1
2 2
0 0
1 1
2 2
0 0
1 1 1
lim [ ] lim [ ]
2 1 2 1
1 2 1 1
lim
2 1
K N K N
K K
k K n k K n
N N
K
n n
Px x n kN x n
K N K N
K
x n x n
K N N


= = = =

= =
| |
= + = =
|
+ +
\ .
+ | |
= =
|
+
\ .



2. (a)
2 2
0
1 1 1 1
lim ( ) lim 1 lim
2 2 2 2
T T
u
T T T T
P u t dt dt T
T T T

= = = =
} }

Como la potencia promedio total de u(t) es finita, su energa es infinita: u(t) es una seal de
potencia.
(b)
2 2
0
1 1 1 1
lim [ ] lim 1 lim
2 1 2 1 2 1 2
N N
u
N N N
n N n
N
P u n
N N N

= =
+
= = = =
+ + +


Como la potencia promedio total de u[n] es finita, su energa es infinita: u[n] es una seal
de potencia.
(c)
2
2 2
0 0
1 1
( )
2 2
at at at
Ex e u t dt e dt e
a a

( = = = =
} }

Como la energa de x(t) es finita, su potencia es cero: x(t) es una seal de energa.
(d)
3 2
2
2 3
0
0
1 1 1 1 1
lim ( ) lim lim lim lim
2 2 2 3 2 3 6
T
T T
x
T T T T T T
T T
P tu t dt t dt t
T T T T

(
= = = = = =
(

} }

Como la potencia de x(t) es infinita, con mayor razn su energa es cero: x(t) no es una
seal de energa ni de potencia.
(e)
2
2
0 0
1 1 4
( 0.5) [ ] 0.5
4 1 1/ 4 3
n
n n
x
n n n
E u n

= = =
| |
= = = = =
|

\ .


Como la energa de x[n] es finita, su potencia es cero: x[n] es una seal de energa.
(f)
( )
2
2
3
0
4 1
1 1
lim 2 lim 2 lim 2
2 1 2 1 2 1
N N
j n
x
N N N
n N n
N
P e
N N N

= =
+
= = = =
+ + +


Como la potencia promedio total de x[n] es finita, su energa es infinita: x[n] es una seal de
potencia
3. Suponga que existen dos representaciones diferentes, x(t) = x
e1
(t)+x
o1
(t) = x
e2
(t)+x
o2
(t). Sean
d
e
(t)=x
e2
(t)-x
e1
(t) y d
o
(t)=x
o2
(t)-x
o1
(t) las diferencias entre ambas seales pares y ambas seales
impares, respectivamente. Entonces x(t) = x
e2
(t)+x
o2
(t) = x
e1
(t)+x
o1
(t) + (d
e
(t) + d
o
(t)) = x(t) +
(d
e
(t) + d
o
(t)), de donde d
e
(t) + d
o
(t) = 0. Pero la nica manera de lograr que la suma de una seal
par y otra impar sea cero es haciendo que ambas seales sean cero. El argumento es idntico en
tiempo discreto.
4. Sean x[n] y y[n] dos seales pares, y sea p[n]=x[n]y[n]
p[-n]=x[-n]y[-n]= x[n]y[n]=p[n].
Sean x[n] y y[n] dos seales impares, y sea p[n]=x[n]y[n]
p[-n] = x[-n]y[-n] = (-x[n])(-y[n]) = p[n].
Sea x[n] una seal par y y[n] una seal impar, y sea p[n]=x[n]y[n]
p[-n] = x[-n]y[-n] = (x[n])(-y[n]) = -p[n].
Universidad Distrital Francisco Jos de Caldas - Anlisis de Seales y Sistemas - Marco A. Alzate 72
5.

6. En el primer caso, la seal decrece a una tasa constante de 1/7 de unidad de amplitud por
segundo, por lo que la seal inversa y los componentes par e impar son como siguen:

En el segundo caso, la seal tiene un forma ar
t
u(1-t). En t=1 la seal toma el valor x(1)=1,
de manera que a=1/r y, por consiguiente, x(t) = r
t-1
u(1-t). En t=0, la seal vale
aproximadamente 0.35, lo que sugiere r ~1/0.35=2.86. Es razonable pensar que se trata de
r=e~2.72, lo cual se confirma al graficar x(t) = e
t-1
u(1-t) y comparar con la grfica entregada
en la tarea.

Los casos tres y cuatro son ms inmediatos:
-5 -4 -3 -2 -1 0 1 2 3 4 5
0
0.5
1
1.5
2
2.5
3
x(t)
-5 -4 -3 -2 -1 0 1 2 3 4 5
0
0.5
1
1.5
2
2.5
3
x(1 - 2t/3)
-25 -20 -15 -10 -5 0 5 10 15 20 25
0
0.5
1
1.5
2
2.5
3
x[n]
-25 -20 -15 -10 -5 0 5 10 15 20 25
0
0.5
1
1.5
2
2.5
3
x[1 - 2n/3]
-6 -4 -2 0 2 4 6
-0.5
0
0.5
1
1.5


x(t)
x(-t)
-6 -4 -2 0 2 4 6
-0.5
0
0.5
1
1.5


x
e
(t)
x
o
(t)
7/7
5/7
3/7
5/7 5/7
3/14
3/14
2/7
-2/7
-3/14
-4 -3 -2 -1 0 1 2 3 4
-0.4
-0.2
0
0.2
0.4
0.6
0.8
1
-4 -3 -2 -1 0 1 2 3 4
-0.4
-0.2
0
0.2
0.4
0.6
0.8
1
exp(-1)
exp(-2)
(1+exp(-2))/2
exp(-2)/2
exp(-1)
(1-exp(-2))/2
-exp(-2)/2
(exp(-2)-1)/2
exp(0)
Universidad Distrital Francisco Jos de Caldas - Anlisis de Seales y Sistemas - Marco A. Alzate 73








-5 -4 -3 -2 -1 0 1 2 3 4 5
-1
-0.5
0
0.5
1
1.5
2


x[n]
x[-n]
-5 -4 -3 -2 -1 0 1 2 3 4 5
-1
-0.5
0
0.5
1
1.5
2


x
e
[n]
x
o
[n]
-6 -4 -2 0 2 4 6
-1
-0.5
0
0.5
1
1.5
2


x[n]
x[-n]
-6 -4 -2 0 2 4 6
-1
-0.5
0
0.5
1
1.5
2


x
e
[n]
x
o
[n]

S-ar putea să vă placă și